You are on page 1of 235
CAREER ENDEAVOUR PUBLICATIONS has taken due care in collecting the data and providing the solutions, before publishing this book. Inspte ofthis, ifany inaccuracy or printing erors are there, CAREER ENDEAVOUR PUBLICATIONS ‘owes noesponsiblly. CAREER ENDEAVOUR PUBLICATIONS willbe gratefulfyou could point ut any such ero. Your suggestions willbe highly appreciated ©All right reserved by CAREER ENDEAVOUR PUBLICATIONS. No part of this book may be reproduced cr utilized in any form without the written permission from the publisher. Preface to the First Edition Quantum Mechanics form an important part of the syllabus for the graduate and post- graduate physics courses of various Indian Universities. While teaching this subject, particularly in physics honours classes, we very often refer to various standard books for the various topics of Quantum Mechanics. But none of these books is fully sufficient for the preparation of different competitive examinations at national and state levels such as CSIR-UGC-NET/JRF, GATE, TIFR, JEST, BARC etc. In all these competitive examinations, the section of Quantum Mechanics catties a high percentage of weightage of the total marks as follows: ‘+ Inevery year CSIR-UGC-NET paper contains questions of at least 35 to 40 marks from this section out 200 marks. + Inevery-year GATE paper contains questions of atleast 12 to 15 marks from this section out 100 marks. ‘The purpose of the present book is to provide a brief discussion of every importanit topic of the Quantum Mechanics syllabus. Each chapter contains a significant number of solved problems some of which contain subtle points, not explicit in the text and’hence will help the students to attain a deeper understanding of the ideas presented in the book. Moreover, the problems that are asked in CSIR-UGC-NET examinations till date, has been solved at the end of each chapter. Besides this, problems that are asked in GATE, TIFR, JEST til date, has been given at the end of each chapter for the practice of the students. This will surely help the student to enhance their self-confidence. Itis hoped that the book will be able to meet up all the requirements of the students preparing for the various competitive examinations: However, any constructive criticism and suggestions for the improvement of the book will always ‘be highly appreciated. Much care has been taken to minimize the typographical errors, yet if any noticed, then your suggestions are invited with a view to improve the book in future editions: The author (Surajit Paul) would like to thank my parents, family members, students, friends and specially Anwesha Chakraborty who have given constant support in the development of the book. Thanks are due to the unit of Career Endeavour Publications for making sincere efforts towards timely publication of the book. Thanks to Mr. Surajit Kuila for his constructive support in doing corrections and proofreading, Last but not the list, a very special thanks to Mohammad Firoz for typing, formatting and designing of the book. CONTENTS CHAPTER 14 12 13 14 CHAPTER 24 22 23 24 25 26 CHAPTER 3a 32 33 34 35 36 CHAPTER. 44 42 CHAPTER Sa 52 83 54 55 : Origin of Quantum Mechanics Introduction Particle aspect of radiation Wave nature of matter Heisenberg’s uncertainty principle CSIR-NET/JRF Previous Years Questions with Solutions Practice Set : Concept of Wave Function Introduction + Physical significance of wave function Hilbert space Operator formalism Schrodinger equation Various representation of wave function CSIR-NET/JRF Previous Years Questions with Solutions Practice Set One-dimensional Potentials Introduction to bound states One-dimensional infinitely deep potential well One-dimensional symmetric finite potential well One-dimensional step potential One-dimensional rectangular potential barrier One-dimensional linear harmonic oscillator CSIR-NET/JRF Previous Years Questions with Solutions Practice Set : Angular momentum & Spin Orbital angular momentum Spin angular momentum CSIR-NET/JRF Previous Years Questions with Solutions Practice Set : Three-dimensional potentials ‘Three-dimensional rectangular box Three-dimensional cubic box Three-dimensional harmonic oscillator Spherically symmetric potentials Hydrogen atom CSIR-NET/JRE Previous Years Questions with Solutions Practice Set 1 14 a 22 24 25 26 27 37. a 44 55 55 64 70 75 80 92 102 110 115 127 133 138 ui 144 147 149 158 164 CHAPTER 6; Identical Particles 61 “62 63 64 Introduction Symmetrization postulate Symmetric and anti-symmetric wave function (space part) ‘Symmetric and anti-symmetric wave function (spin part) CSIR-NET/JRF Previous Years Questions with Solutions Practice Set CHAPTER 7: Perturbation Theory 74 72 ‘Time independent perturbation theory Time dependent perturbation theory CSIR-NET/JRF Previous Years Questions with Solutions Practice Set CHAPTER 8: Scattering Theory a 82 83 84 8s Introduction Scattering cross-section Scattering amplitude First Born approximation Partial wave analysis CSIR-NET/JRE Previous Years Questions with Solutions Practice Set CHAPTER 9: Variational and WKB method on 92 Variational method WEB approximation (Wentzel-Kramers-Brilouin Method) CSIR-NET/JRF Previous Years Questions with Solutions 169 170 171 172 176 178 179 192 198 206 209 209 210 214 213, 216 218 220 224 227 Origin of Quantum Mechanics 1.1 Introduction : Atthe end of the 19th century, physics consisted essentially of Newton’s Classical mechanics and Niaxwell’s theory of electromagnetism. Classical mechanics was used to predict the dynamics of material bodies and ‘Maxwell's electromagnetism provided the proper frame work to study radiation. In fact, matter and radiation were described in terms of particles and waves andthe interactions between matter and radiation were well explained by Lorentz, phenomena can be & Inthe 20th century, fon and its surroundings takes place een the em. wave of frequanecy of energy. for light and| posted that light microscopic level. n SHE Fadiation exhibits particlelike behaviour but conversely’ materi Te CUT ERS PMc was experimentally confirmed by Davisson and Germer in I gare nde i Generally there are two independent formulations of quantum mechanics. The firs formulation called matrix mechanics, was devoloped by Heisenberg in 1925 and second formulation called wave mechanics, was: devoloped by Schrodinger in 1926, which is the generalization of De-broglie postulate, These two different formulations of quantum mechanics were shown to be equivalent. Moreover, Ditac has suggested a more general formulation of quantum meckanics which deals with abstract objects like Kets (state vectors), bras and operators, The representation of Dirac formalism in a continuous basis i. in the position and mo- mentum representation gives back the Schrodinger’s wave mechanics and the representation of Dirac formalism in a discrete basis gives the Heisenberg’s matrix formulation. 1.2 Particle aspect of radiation : According to classical physics, a particle is characterized by an energy E and a momentum P ,whereas a wave is characterized by an amplitude and wave vector & that specifies the direction of the propaga- tion of wave. For instance, the particle and wave properties are mutually exclusive and wave can ex- change any amount of energy with particles. Now, we will ee that how the phenomenas like phatoclec- Origin of Quantum Mechanice Photoelectric Effect: During his experiments on e.m. waves in 1887, Hertz noticed that sparks occurred more rapidly in the air ‘gap of the transmitter when an ultraviolet light is directed at one of the metal balls used in that experiment, Phillip Lenard, a student of Hertz studied the phenomena, He soon discovered that when the frequency of light is sufficient'y high, electrons are emitted from the metal surface. ‘The phenomena in which electrons are emitted from the metal surface (e.g. Na, K, Csetc,) when iradiated with light (photon) is called “Photo-electric effect”. The metals which exhibits this phenomena are called “Photo-sensitive materials” and the emitted electrons are called “Photo-electrons”. The existence ofphoto electric effect is not surprising. Light waves carry energy and some of the energy absorbed by the metal gets concentrated on a particular electron and reappear as their kinetic energy. Lenard’s Experiment : Lenard's experimental arrangement is as shown in figure above. Two plane metal plates C and D are ‘mounted in an evacuated tube. Plate G is coated with photosensitive metal and maintained at a negative whereas A another metal plate and kept at zero potential, The beam of negative ions emitted from C can pass through a hole in A, UV-radiation is incident on C through the quartz window W fitted to the side tube G and negative ions are emitted These are attracted to Aad a part ofit passing through the hole in Aiscollated at D. These Rigctrometer E, where, m is the mass Further, eponiepa a ale is foupdito decrease with theincrease of positive potential UINAANEMs Sa ,) the current becomes zero. Then, this V, is called stopping potential, Electrons of al velocities up to the maximum v, are stopped fromteaching 4. Then Some experimental observations: ° > v ° vo Figure : Variation of photo current Figure : Variation of photo current (@) with the intensity of radiation (1) (i) with Potential difference (V) nice heair acyof sated zalled photo metal Dare ative Cean eside olein sseof zero. "ped Origin of Quantum Mechanics ee _ at an Vs; Ven Vs, ae Ye Ym My V Figure : Plot of photocurrent (i) against Potential : difference V showing the variation of stopping Figure: Plot of E,,., against v. ‘potential (V,) with frequency () of radiation. (There is a certain minimum frequency v, of incident radiation below which there is no emission of photo-electrons regardless of the intensity of the incident radiation. This frequency is known as threshold frequency which depends on the material and the nature of the emitting surface, (i) The maximum velocity or kinetic energy of the emitted photoelectrons depends on the frequency of the incident light, not on it's itensity i.e, blue light results in faster electrons thanred light. The kinetic energy of the ejected electron increases linearly with incident frequency. (ii) The emission of photo-electrons i instantaneous i.e. no time interval between arrival of light at the é i le the incident light is, (iv) Ata fixed freqie i j i = the intensity ofthe light but ( According to of threshold f L a Gi According to wale g uted over the entire wave front, frequency ofllight, no concept So, electrons w cept of instantaneous emission. (iii) The electric fi E {forcesta liberate clectrons from the metal surfag er fields and should give higher velocity to the ' Einstein’s hypot! anta of energy ¢ = hy. ficentrated in small packets known the whole energy to one electron ii) Energy in light i as ‘photons’. 7 without any time’ (iii) A certain: Laaaaae a eae to release an electron from the metal surface Photo-clectric equation: When a radiation of frequency v isincident on the metal surface in vaccum, it canbe either be reflected from the metal surface or it can be completely absorbed by an electron. Inthe second case, the electron can be knocked out of the surface. The emission of electron from the metal surface is possible only if hy > Wy. The remaining energy (hv-Wy) canbe utilized to impart kinetic energy ofthe emitted electrons. Figure: Mechanism of Photoelectric effect. Soln. Soln, Soln. iS Origin of Quantum Mechanics So, the maximum Kinetic energy of the emitted electrons is 1 Ex guax = MV aay, = AV — We = hv — ho 2 where ‘m’ is the rest mass of the electron, v.q. is the maximum velocity of the emitted electrons and V, is the threshold frequency. This is the well-known Einstein's photo-electric equation, Explaination of Photoelectric effect: (i Ifv Ag = 75904 Example3. The threshold wavelength for photoelectric emission in tungsten is 230 nm. What wavelength of incident light must be used in order to eject electrons with a maxinmum velocity of 510° mis [Mass of the electron= 9.11073 kg] Fromthe photoelectric equation, 10 a nics Origin of Quantum Mechanics as aoe 10726 BSE = DESO 413.7541" - 2 =0.0203%10°S ‘Therefore, the required wavelength = 2030A. aa Example 4. The work function of aluminium is4.2 eV. Caleulae the kinetic energy of the fastest and slowest photoelectrons, the stopping potential and the cut-off wavelength when light of wavelength 2000A fals ona clean aluminium surface. atno Soln. According to Einstein's photoelectric equation, the KE of the fastest electrons given by —— Eg max = hv -Wo= (6.2 - 4.2) eV=20V “The kinetic energy ofthe slowest photoelectrons is zero, _ Now, the stopping potential is equal to the maximum KE of the photoelectrons, i. 2.0 eV. 19 x ABx1 6x10 Fe raxtolS fs ind is sed from the surface of lithium by where, Aisa consta jon of lithiumis 2.4 eV Soln. Here, E = The kinetic energy the frequency ofthe incident light. 7 Therefore, the ee ‘The maximum KE ; | Example 6 Ulravige ie is directed ata potassium surface. 10.50% of the invidegty HSWHLY BlofSelectrons are emitted per second if the potassium surface WP Soln. Energy ofthe single AAPA, ‘gareerendeavour. com aS 3S eV = 5.6810 0-350 ‘Therefore, the number of photons that reach the surface per seconds, Power _ Intensity xArea _ (1-001 /m?)x(1.00%10~4 m?) Ey Ey 568x109 ‘Therefore, the rate of emission ofphotoclectrons is, Hhof 1, =(0.0050)n, = 8.810"! photoelectron/s ‘tthe Example 7. Calculate the magnitude of the linear momentum ofa photon of radiation having electric field described by E = Epi cos[ ko (»+2)—ar]. Soln, The equation forthe electric field ofan electromagnetic wave can be written as ‘9008 (£7 ot) LSI Origin of Quantum Mechanice Soln, Soln. Soin. Thus, & = koj-+ kok and 7 = xi+yp+zk ‘The finear momentum of the photon is B= hk = (kaj + kok) ‘The magnitude of the linear momentum is l= ig =v Example 8. Calculate the magnitude of the linear momentum of a photon in a beam of He-Ne laser (A= 634 nm) in the units of oe ic 1240eV nm PET Fe = (B4nm)c 196 Vic Example 9, Abeam of light of wavelength 420 nm has the same intensity as another beam of light of wave- Yength 630 nm, The first beam, fling perpendicularly on a certain area ofa wall, deposits 12x10" photons per \thesecond beam deposit per secondper square centimetre of: The intensity of a wa a unit time on a perpendicular cross-section, Thus bath syd if 1e one square centimeter area first beam the corresponding Example 10. Beam of pq eeargerendearonr 0 rion 2.0 eV placed ina magnetic field B, The most energetic el Apantbeareerent licular to the field, ace bent in ciccular arc ofradius of 20 ‘cm. Find the value of B, Kinetic energy ofthe most energetic electrons is Bema, AE Hg AOA 2 007 =0.16e¥ = 12x18) Linear momentum of the most energetic electrons = mv = /ZmK = 4.671075 kg mis ‘When a charged particle is sent perpendicular to a magnetic field, it goes along acircle ofradius 4.67 x10 kg mis 610°C) x (0.20m) re Ba Be qB qr A6x10ST e laser iwave- onsper ondper dicular ermea onding ignetic sof20 Origin of Quantum Mechanics Compton Effect: ‘When a beam of monochromatic X-rays of wavelength 4 is scattered by a light element (e.g, carbon), it is observed that scattered X-rays have maximum intensities at two wavelengths 2 and 2 (4' > 2).The radiation at wavelength 4 is called unmodified radiation whereas radiation 2 at wavelerigth is called modified radiation. This phenomenon is known as Compton Effectand the wavelength difference A’ =A is called Compton shift. This shift is independent of wavelength 4 of the incident beam and the nature of the scatterer but depends on the angle of scattering. Experiment: ‘A beam of monochromatic X-rays is allowed to fall ona block of graphite and the intensities of thescatiered X-rays are measured as a function of wavelength of X-rays, at different scattering angles, gyObserver Foreach angle of sc fo aiprent wavelengths, Figure : Variation ofintensitis of scattered light as a function of wavelength for various scattering angles (#). ae Origin of Quantum Mechanics Failure of classical theory: Inclassical wave theory, X-rays aree.m. waves. If v is the frequency of the incident X-rays on the material, then they cause electrons in the material to oscillate at the same frequency v. Hence, the scattered X- radiation should consist of single frequencyv (or single wavelength 2 ). gain, electrons should radiate X-rays uniformly in all directions and hence, wavelength of the scattered radiation should not vary with scattering angle. Assumptions: Compton applied Binstein’s quantum theory of light with that assumption that the incident photons posses ‘momentum in addition to energy. 1, The beam of incident monochromatic X-rays of frequency v consists of streams of photons each of h y and momentum P=""”, traveling along the direction ofthe beam with velocity ‘c. 2. The scattering is due to the elastic collision between photon and the five electron at rest. Hence, the conservation laws of energy and momentum hold good in the scattering process. energy ¢ Explanation: We considera beam of monochromatic X-rays offiequency v and wavelength 2 beincident on target electron. Let, the incident: : angle ¢ and the electroff Ifthe frequency ofthe: Therefore, the kinetic ef wunvcaresrendeavour ct com Vi-@/e ~Q) Applying law of conservation of energy: hv = hv’ + E, Applying law of conservationof tinear momentum along and perpendicular to the direction of incidence: ~Q) From the equation (3) we can obtain the compton shift to be, h cazat=—L(1-cosp)=a,(I-e A-d=Ah at cos) = A, (I-cos¢) @ 02496 4° i¢Comnton wavelenoth where. 2 ea wnics Origin of Quantum Mechanics MES @ Conclusions: terial, (® Compton shiftis independent of the wavelength of incident radiation and of the nature of the scattering dX material. “rays (i) Compton shift depends on the scattering angle @. tering (Gi) Largeras the rest mass of the scatterer is smaller. Relation between vandv': “ =1+—" (1 cos¢)=1+a(1~cos¢) posses v mee’ ach of Relation between @ and g: cot =(I+a) tan’ Kinetic energy of the recoil electron: 2e, the target length. aghan 1ap the intensities of unmodified X-rays Soln. sively, Son. ‘Example 12. A beam of 7 -radiation having photon of energy 510 keV is incident on a foil of aluminium. Calculate the wavelenth of radiation at 90° and also the energy and direction of the emission of the corresponding electron. he Energy of a photon of frequency vis €= hv a Here, e= 510 keV =510x10° x1.6x10-9 7 he __ 66x10" x3x108 € 510x10°x1.6x10- The wavelength ofthe scattered radiation is, a =2.426x10°? m Origin of Quantum Mechanice Ori gee rie a+ (-cosg) 2.426 x107!* +6.6x10 Uses) moc 9.1x107 9! x3x3x10 = 2.426% 107)? + 2.417107? = 4.843% 1071? m The energy of the recoil electron is, B=hv-hv'en nv) = AAD 6.6010" x3%108 x (4.843% 1071? —2.426x10""*) 2.426%10"}? x 4.84310" ‘The direction of the recoil electrons is given by, 72x10 47 cot 45° 1 1 140.9985 1.9985 6.61074 Soln. —— 14. Aphoton of energy ¢ is scaticl ‘a free electron of mass m, and observed at scattering Ta éareerendeavourcom angles greater than %. Show tat the energy €" ofthe scattered photons ks than 2mgc?, however large the value of € maybe. Soln. ‘The energy of the scattered photon is, e e 1+2asin?® 142 sin? S 2 met 2 28 x For scattering angle ¢> pans #oSsintg>tat Therefore, the energy ofthe scattered photon is, > das sing is an increasing function of g. For any value of = where 421 according as @2 7. anice Origin of Quantum Mechanics ttering ‘eethe lue of = € € 2mige” —s—- = Lt 26 Eee meg mye? 4 2nige’ E : 7 mpc? Since, positive quantity and f > 1, “0+ isaways greater than {and ¢ isabvayslessthan 2mgc? , however large the value of ¢ maybe. 1.3. Wave Nature of Matter: According to classical mechanics, the particle can behave asa particle and wave can behave like a wave. Particle is characterized by energy and momentum and wave is characterized byamplinide and wave vector. In 1923, De-broglie postulated that a moving particle can behave as a wave. The de-broglie wavelength associated with a particle of est mass ‘m,’ moving witha velocity ‘v’ is given by is Lorentz factor 927. Ifthe De-broglie wavekngth can be experimentally detected. Otherwise, ifthe Dey sl ot be detected experimentally Schrodinger modi a group of waves Phase Velocity: where, o isthea ‘The phase ¢ ofthe wave atany tine e ieanens #(3)= aaa emer te WWMNSGATEPLRNGEAVOUL COM. gcocesmrersos also the velocity of a monochromatic wave with which crest or trough of the wave travels ina medium Ina dispersive medium, ae isdifferent for different wavelengths. The phase velocity of the propagating monochromatic ere aveis givenby, Yp ==. waveis gi ean From the relativistic mass energy relation, y, == =e _ ¢ “Pp ymy ov Since, the phase velocity of'a de-broglie wave is always greater than the velocity of light in freespace. But according to the special theory ofrelaticvity, no energy can be transmitted with velocity greater than ¢. This suggests that the phase velocity has no physical significance and We can not respresent the wave nature of moving particle by a single plane wave. Origin of Quantum Mechanics Group velocity: os] A wave group or wave packet is described as the superposition of individual waves of different wavelengths These waves interfere with each other and results in variation of amplitude, These defines the wave group shape. If the medium is non-dispersive, then phase velocity of the individual waves will not change with wavelength. Asa result, wave packet travels with common phase velocity. But, ina dispersive medium, phase velocity of individual waves will be different. As a result, wave packet moves with a different velocity thax phase velocity which is known as group velocity ‘,’. Let, the wave group is formed by the superposition oftwo monochromatic waves having same amplitude but v5 < vp av, (@Anamolous dispersive medium: “7 =-ve=> v, >, Example 15. What Voltage must be applied to an electron microscope to produce electrons ofwavelength 0.40A? der a potential of V votsis, 1 Thede-Broglie wavelength 2 ofaparticle is 4 oc —L. and fora photon 4% {F Thisshows that de-Broglie VE wavelength of an electron is much smaller as compared to that ofa photon for the same energy E. Example 18. Energy of a particle at absolute temperature T is of the order of kT. Calculate the wavelength of thermal neutrons at 27°C. [Given, the mass of the neutron =1.67%10-*4 gm. Planck’s constant, k=6.67x10" erg sec and Boltzman constang, k =8.6x10~> eVk] ‘De-Broglie wavelength of the neutron is h h Az = t= =1:781° V2mE V2mkT Soin. Origin of Quantum Mechanics Example 19. Abeam of monenergetic neutrons corresponding to 27°C is allowed to fallona crystal, A first order reflection is observed at a glancing angle 30°. Calculate the interplanar spacing of the crystal, Given, Planck constant, h= 662x104 Js Mass of neutron, m=1.67x1077 kg Boltzmannconstant, — k=1.38%10 J/K FromBragg law of diffraction we have, where, the glancing angle, = 30°, order number of diffraction, n= 1 2d sin30°= A, or, 2d x. ,or,d=d 2 De-Broglie wavelength ofthe neutron is h h de ZO\I MeV using both relativistic and relativistic expression for 4? 14. Hoisenberg’s WMA TeRERNGeAVvoUr.cOM According to classical mechanics, a moving particle has a definite momentum and position in space. Both the position and momentum of such a particle can be definitely determined. Recently, the classical viewhas been proved to bean approximation and is inadquate for describing the dynamical behaviour of the macroscoy objects. It cannot describe satisfactorily the behaviour of sub-atomic particles. In quantum mechanics, a particle is described by a wave packet of linear extension (Ax) which surrounds the position ofthe classical particle and moves with grou velocity. According to Max Bomn’s probability interpre- tation, the particle may be found everywhere within the wave packet. This means that the position of the particle is uncertain within the limits (Ax) ofthe wave packet. So, we may say that the particle may be found anywhere within (Ax). Again, a wave packet is formed by the superposition of large number of waves having different wavelengths, the wavelength the wave packet cannot be said definitely. What we can say that the wavelength ofthe wave packet lies within certaif range 4 to 4+ .Asthe momentum of a particleis related to wavelength by i 1 { ; os first | Origin of Quantum Mechanics = ie) h Gaal AA the particle momentumis also withinp to p+ Ap . So, we cannot definitely say what is the momentum of the particle. Hence, because of the wave nature of pastice there is uncertainty (Ax) in position of the particle as well as uncertainty in corresponding momentum (Ap) . Ifthe number of superposed wave as well as their range of wavelength (A2) be increased, then the linear speed (Ax) ofthe wave packet decreases. But simultaneously the uncertainty (Ap) in momentum increases as Ap x AZ. When AA ==, Ap =o and Ar =0, ie, the wave pocket reduces to a point. Inthis position, there is no uncertainty in the position of the particle. Now foradefinite 2=0, A =0, then Ap=0(-" Ap o AZ) ,thenthere isno uncertainty inthe momentum of the particle but in this case Ax =00, ie, the position uncertainty is infinite since the associated wave with the particle may now extend up to «o. Statement of the wiée The uncertainty p Je sumultancously the position action. 104 Js. Here Ax is the error or (uncertainty) inthe determina fely then Ax becomes smaller and ertaintes arenot inthe appratus ps TE © Wax=0, the. Wnnascat SeferideavOur.co This means that, ifthe uncertainty inthe position measurement of a moving particle be zero, then atthe same moment the uncertainty in the momentum measurement willbe infinity. (b) If the momentum ofa moving particle at any instant be determined accurately (1¢., Ap, = 0), then the uncertainty (error) in the determination of position of the particle will be infinite. (©) Fora particle of mass ‘m’ moving with velocity v, along the x-direction, its momentumis p, =mv,, For aheavy particle mp andin that case, AxAv, For sucha patticle both the position and momentum can be determined accurately. Thisis true for macroscopic bodies whose motions are discussed in classical mechanics. Thus the uncertainty relation has no meaning in classical mechanics, But for microscopic objects such as electron, proton, atom, etc, quantum mechanics is annlicahle and acsnch the nncertaintvrlatinn ic reality in deccrihina the damaminal hehavinwrafench ahiacte q@ Origin of Quantum Mechanics ‘Time-Energy Uncertainty Relation: ‘The time-energy uncertainty principle is stated as follows: Inany simultaneous determination of time and energy of a moving particle, the product of the uncertain- ties is equal to the Planck’s constant, AEAt>h Here, AF is the uncertainty in the measurement of energy of Ar, the corresponding uncertainty in the measurement of time. If AE be the maximum uncertainty in the determination energy of a system in a particular state,then A isthe ‘minimum, ie., the system remains for a minimum time in the state. Qn the other hand, ifa system remains ina particular state for a maxinium interval oftime, then the uncertainty in the measurement of energy s the mini- mum, Angular momentum- angular position Uncertainty Relation: ALAG>h Where, AL is the uncertainty in the angular momentum ofthe particle and Ag isthe uncertainty in the angular position of the particle agi Causality in Quantufh S166 instant is sufficient for the prediction ofits fut assical principle that, the future behaviour ofa particle at a given instant. The law of} causality, however, hol Sn be made without disturbing a microsystem, it does According to Max /0 alternate angles. In wave- described by a wave function e behaviour of the system ata | point P(7) at any tis shows that even for cannot be carried out system unambiguousy fl die 0. Thus the principle of eatisa fe Therefore, the pro i¢ interpretation of the observed | events are tobe adopted in accordance with the unc: WWW.CaréerendeaVour.com | ‘view of uncertainty principle: Concept of Bohr Orbi Wehave the energy-time uncertainty relation, AE.At> A According tothe concept of Bohr, each electron revolves around the nucleus in quantised stationary orbits with definite energy E. So, the uncertaintyis AE =0. is aE 0 Hence, all energy states of the Bohr atom must have infinite lifetime (r =o) . This implied that radiation | So, would not occur due to the transition of electrons, But actually A¢~ 10~® sec leading to the occurrence of | emission of radiation due to electronic transition. Thus the concept of Bohr orbit violets the Heisenberg’s uncertainty principle. i cs ith’ Origin of Quantum Mechanics el G@ h Again, if At is finite, then AE = is also finite, This means thatthe energy levels in the excited state must have a finite spread. As a result the radiation emitted due to the transition ofan electron fromthe excited state to the ground state is not monochromatic. This leads to the conclusion that the spectral lines can never be ‘infinitely sharp but must have a natural broadening. This is contradiction with Bokr’s theory of spectral series, Electron difraction through a single slit: Let a monoenergetic parallel beam of electrons moving along the x-direction be incident ona single split of width Ay. As the electron in motion is associated with de-Broglie wave, the electron beam after passing through the slit undergoes dftaction producing a raunhofer diffraction pattem onthe photaplateperpendicular to the x-axis, é shebeerainty inthe position ofanelectron intheslitis Ay asit Td Enh gélegtron in the slit, CAREER ENTER Before incidence, AWWRAAM BARONE MAGA NORM UGG tion be p. Aer diftation ifthe electron moves along OC, then the component of momentum p along the y-direction is p sin @ . Since the electron may be anywhere within -@ to +6, the y-component of momentum must be anywhere between =psind to psind. ‘Thus the uncertainty in the momentum measurement of the electron is, dpy= psind~(-psind)* 2psind =24sin 4th Therefore, AY-Apy = ax To decrease the uncertainty inthe determination position of the electron, Ay must be decreased. But the above equation shows that a decrease in Ay leads to the increase in g and the diffraction pattern will be wider. This will cause a larger uncertainty in Ap . Thus the electron diffaction experiment established the truth of uncertainty principle. sind =2h2h Soln, Soln. Origin of Quantum Mechanics ‘Consequences of Heisenberg’s Uncertainty principle: (1) Non-existance of electrons within the nucleus: Suppose electron exist within the nucleus, then uncertainty ih the position of nucleus willbe ofthe order of twice ofradius ofthe nucleusi.e, Ax =2x10"m. ‘Then the minimum energy ofthe eleetronis given by Eh, = piel tie! = Ey = (Ap) c? + mic* a = Bay = (4) ct +mic! ~10MeY 2a Therefore, ifelectron exist within the nucleus, then minimum energy of the electron will be approximately 10 Mev. But, experimentally energy comes out to be of the order of 4 MeV. Thus, we can say that electron does not exist within the nucleus. (2) Ground state energy of a linear harmonic oscillator: Leta particle ofmass mis executing SHM along x-axis and let at any t position, Therefore, i Total energy of LHO ‘t itspositionis‘x’ fomthe mean For minimum energy, ‘ni il So, the minimum ener [7 Merrie parvo meeiinies poe snand momentum] a_ WWW Careernt deavour.com ma my ie mt On my ‘Example 22. Suppose that the momentum ofa certain particle can be measured to an aecuracy of one part in 1000. Determine the minimum uncertainty inthe position ofthe particle ifthe particle is ()a 5x10 4g mass moving witha speed of 2 m/s; (i) an elecron moving with a speed of 1.8108 m/s Given: 222107 = ap =107%p=10°3 mv P Therefore, the minimum uncertainty in position ofthe particle will be Origin of Quantum Mechanice h y-20 40 ) (AX) pg => = 5.2810 Onin 4nmgyxl0> hyl-v/e? i) (Ax) nig = @ (A) nia 4mmgvx10> Example 23. The average time interval that elapses between the excitation ofan atom and the time it radiates 710 energy is 19° sec. Calculate the uncertainty in the energy of emitted photons and the limit of accuracy with baad which the frequency of the emitted radiation may be determined. Soln. Uncertainty in energy of the emitted photons willbe hagtaxioev ar ean Soin. th, . ou Example 25. In fatom is produced by vapourising silver salt in a furna jnty principle to find the order of magnitude of the sme Shee Phos om al redre esverirebit 107 7K Mass of yeogen atom = 166x107” kg ] Soln. Ifthe silver atom of mass ‘m' moves with velocity ‘v' along the x-axis from the furnace at temperature T, then its kinetic energy is, mv? ser tin m = Time required by the silver atom to travel a distance of 100 emis, I 100 i | BIT im | Now time-energy uncertainty relation is, AEAt =A => (AE) iin (Mtn 2A => (AE) cin ® rin Soln. Soln, Origin of Quantum Mechanics wow (Alan? = AE ap = mg = 2 ‘Maximum uncertainty inthe position of the particle 7, ag = 7107 m = 0.7m 1. [Be arr (#) The smallest spot on the detector corresponds to the maximum uncertainty in position ofthe silver atom. Hence, size of the smallest spot=0.7 nm, Example 26. The speedometer ofa 180 kg vehicle reads 72 knw/h ata particular instant, The readings are ‘known to have an accuracy of +2 knv'a. What would be the minimum uncertainty in the position ofthe vehicle at that instant avcotding to the Heisenberg uncertainty perinciple? ‘The uncertainty in the momentumis Aps = A(mv,) = may, Thus the minimum wi Remember the size of snatem is off Opa faster is 10m Asize of 1 mistoo Areit. ; Fe nuclei make transition from is about 3104s. ifthe spread in vacuum, From energy-time 2 | Origin of Quantum Mechanice @ CSIR PREVIOUS YEAR QUESTIONS Qui. Which of the graphs below gives the correct qualitative behavior ofthe energy density ,(2) ofblackbody radiation of wavelength 2 attwo temperatures T, and T;(T, Z,). The energy emitted per unit time per unit solid angle per unit surface area of a blackbody in the frequency range to v+ dy is given by B(v)dv. Which ‘one of the following graphs has the correct form? [TIER 2015] BG)| u BO) that @ ® > 0 v 0 v esa ee Bo) © 0 n. wed www.careerendeavour.com Concept of Wave Function Soln. 2.1 Introduction : There is always a quantity asscociated with any type of waves, which varies periodically with space and time, In water waves, the quantity that varies periodically isthe height of the water surface and in light waves, electric field and magnetic field vary with space and time. For De-broglie waves or matter waves asscociated wat moving itl the quay that vary wih space tne, 3 ‘scalled wave function of the particle, eee finetion y-(348) ) F should be cong pywhere ih pete Butea ous in some cases as follows: {@) Ifthe potential (b) If the potential un ‘of discontinuity at some points, (iii) v(x.) should be; Example 1: Which of for all values of 2? (@) y(2)= Aseox www cateerendeawodr.comy «)=4 y (x)= dsecx and y(x)= Atanx isnot finite at x=, ution of the Schrodinger equation w(x)=de™ isnotfinteat x= 40; y(x)= Ae isfinite everywhere in space Correct option Is (d) Example 2: Aball bounces back offearth. You are asked to solve this quantum mechanically assuming the earth is an infirtely tard sphere. Consider surface of earth as the origin implying V (0) =<» and a linear potential eleswhere (ic. V (x) = -mgx for x > 0). Which of the following wave function is physically admis- sible for this problem (With & > 0): (b) v(x) = de’ ey (x)=naee (8) v(x) = Are b ws: ats, Concept of wave function ! Soln. ie) Since the earth has been taken as infinitely hard sphere, therefore the wave function of the ball willbe zeto for <0, non-zero forx>0 and zero at infinity. Since y (x) should be continous everywhere, the wavefunction of the ball will be zero at x=0. Correct option is (d). 2.2 Physical significance of wave function : Generally, y (x,4) is a complex quantity. It can be mutipliedd by any complex number without affecting its physical significance. In general, y(x,f) has no direct physical significance. But the quantity v' («ty (2.0) =|y(x.0)P is teal, physically significant and is defined as position probability density ie. probability of finding the particle per unit length at time ”. Therefore, for 1-D motion the probsbility of finding the particle between x to x'+ dr at time ‘? is given by and for 3-D motion the] 3 tinge element dr located between F and? +dF at ting Now, v(xA)or vl space should be eq This can be understood as a ‘Method of Noman careerendeavour. com Consider (x,t) is an unnormalized wave function. We can construct a normalized wave function as = N@(x,f) where Nis the normalization constant, Therefore, (ue Jo (tw thee=|N4|f 9 (x)o(athir=1 Jo edeCne Example 3: Normalize the wave function given by v(x)=Ne* (x>0) =Ne* (x<0) =>N Sol. Normalization condition is j v'yde=1=>|NP i enacefea| = a Concept of wave function 1 N= oi [ae tag)" = Orthogonality condition of wave functions : ‘Two wavefunctions y,, (x) and y, (x) are said to be orthogonal to each other, if Ivete Wva(x)de=0(men) i.e. if'a particle isin the state Y, (x), then the particle cannot be in the state y, (xx) simultaneously together. Orthonormality condition of wave functions: Ina vectorspace it pl basis ie. we can express any ‘vector inthis vector sp ined space can be defined in which a set of fiunctions ¢ (x) a coordinate system. The corre- sponding infinite dimets ed inthe interval ax [8 Be] =ih Similarly, [J,A, ]= it and [2,3,]=ih Linear Operator: Tfan operator A is said to be a linear if (0 ALev(x)}=edy (x) and (i) Alyy (x) +¥2 (x)] = Av, (x) + Ay (x) ‘All Quantum mechanical Operators are linear mnature. Properties of the commutator bracket: [: {8.4} 2.{. ac a Concept of wave function Soln: Soln: Soln: Solu: 4.[48¢]-[4 8]é+ 8] 42] 1.[7(4).6(4)]=0 10.[ 4",8]=n4™"[4,8] Eran Find te commmtatrbnte 8 4,8) va [staal Lanka (aagas x 2in(xp-+ px) [27] e+ [»[sa]]- Js Ay Eigenvalues and Fig} Ian operator § CAREER ENDEAY OUR, J), tno v, (0) is cated te cigenfiunction of A corres eae above pions mas eigenvalue equation not A ORR CAPT SAGGANOUI CO ofoperator A, Brample: d= 2°, and y, (2)=a0™ = Ay, (2) -£ (a) = dae =4y, (2) Ve r isaneigenfunction of A corresponding to eigenvalue 4. Ifthe commutator bracket of two operators A and B is equal to zero ice. [ A, 3] =0, then the physical observables corresponding to these operators are simultaneously accurately measurable and they havea com- plete sef of simultaneous eigenfianctions. a Example 7: The operator (« + 4) has the eigenvalue or . Determine the corresponding wavefunction. tigenabccsatin: (+L Jy = oy = Wx (a-s)y = [SE flax) stlon vd the sation ysical Concept of wave function Jeinyy-sy =yoeuo{ x2) ‘Example 8: Find the constant B which makes e"*"’ an eigenfunction of the operator ( ‘What is the corresponding eigenvalue? a 5 For e* tobe eigenfithétion of the operator ( — Bx? | , then the eigenvatue J (4a*x?~2a~ Bx?) mustbe independent of x ie. (4a*-B)=0= B=4a* 2 Taco —~By = ie rator is (-2a). Schrodinger represdhage 4) Hyde Properties: (1) Bigenvalues ofh maaan a ‘cdniit ve Hence, the momentum operator , ishermitian in nature. 1 Similaiy, we can see that — isant-hermitian ie, [2] 2 ox a. Projection operator: A operator is said to be projection operatorif A' = A and 4? = A Properties: (1) Eigenvalues of projection operator is 0 and 1. (2) Product of two projection operators is also a projection operator if [6,, 5.]=0 G) Sum of two projection operator is a projection operator if { f,, 2} =0 Concept of wave function Soln, Unitary Operator: Anoperator / is said to be unitaryif 4' = 4"! Parity Opertor: Parity opeartor corresponds to space reflection about the origin ie hy (x) Ingeneral, Py (*)=y (7); where 7 = xi +y}+ 2 and - When parity operator acts ona wavefunction, the following changes take place in various co-ordinatesystem: () Cartesian co-ordinate system: x > —x, y > -y,2 9-2 (i Spherical polar co-ordinate system: r+ 7,09 x-0,>2 + (i) Cyllindrical co-ordiante system: p> p,g>2+$, 2 -z Properties: (1) Eigenvalues of the parity operator is 1, -I Pu (x)=v(-x)=w(x)—v even parity. By (x) =y (-x) =- (x) > odd parity Q) Parity operator is hermitian in nature ) Party operator commutes with anton oper ifthe potential under which particle is movingie. V(x) issymmetrc in nat a —— Example 9: Let, N then eigenvalues of N & (a) +veand -ve integets Example 10: Let the ‘What will be the functipg vse” | GAREER, ENDEAVOUR, "www. cateérendeavour.com te Pa) Wed [24 ota? 2m dx| a? ‘This isnot an eigenfunction ofkinetic energy. Example 11: An operator Ais definedas Ay (x)=y" (x)y (x)-Isthis a linear operator? Soln. Forthe given operator, Alyi(x)+v2(x)]=[vi (3) +¥2() [a )+¥2(0)] -vi@)+v5@) [m+] =w (Dn (x)+ y's (dvr (2) +¥s Cedys (x+y (v2) zion} Concept of wave function = and Ay (x)+ dyo(x)=¥'i (xvi (x)+¥"2 (xSvu(a) Tus, [4 (2) +2 (x) =v" (x) + Ay (x) and hence A isnot a linear operator. Example 42. Prove that [py H]= 2p +x [PY] where Hao ay, and V is potential energy stem: operator. y 2 = 7 = H\p, = Pray ese Sol. [Xp,, H]=X[py, H]+[X, H] py A pode peter, =X [pee ealX, Pel+LXs re] Pe} Ps 7) 4 (24 ih =X [pal ]+ 5, (2itipr) Pe =o +X[p,¥] Ve) Example 13.|¢,) and |g.) be twe ortonormal state vectors. Let A=|¢))(2|+|¢) (| - Is a projection operator? e : 0; Sola. 4* =[|)(¢,|+ |=4 Hence, Ais hermit ‘Therefore, Aisnot of 7 Example 14. Propet Soln, Gj = Syn 7) = (Pa? 2 J+.) (2inX) +(2inX) p,) a Hip,X+ Xp.) = 2 (Abe + PX) "Sin P= Example 15, Prove thatthe operators i(d / dx) and d? / dx? are Hermitian False soe ty Sut le y Soin. Lv. eg )vsteaavivarsf ve Sit ~Flege.) vide Therefore, id /de is Hermitian. - dy, J _ dv, dv, jy Ml fave dve gy ly. al Vee Soln, Soln. Soln. a5 a= fo Oey ve] +]v.4 gee Neat de Thus, d? /dx” is Hermitian. Example 16: Find the following commutation relations: ae [23 «|Z Fe] & (alae s1Ona On) (0% -2) = Ola 7 "oe | laat wa)" lar ar |” 6. Fy Oy _aF oy Eye pe ee (ole ora on gant Gti) (2 ae e Thus, [2 -F0)|-% Example 17: Find th olde & a Therefore, : x > oes REER‘EA iv. Caréerendeavour.com Example 18, By what factors do the operators (x" p? + p?x*) and 1/2(xp, + p.x) differ? (ple pie) f a a Of PED | wo Sint ( 4 J _ = ( 2o Sow oals “| @ Concept of wave function lon — | concept of wave function Soln, = ~ hon, spake 4) Aldea] oP (pF ae xo 22S x, = (anSt prorSaeT ra Es ay 7 ah ef a(t F 44x 4h = {me Fare, ‘> Bax Se tareasls The two operators differ by a term ~(3/2)h? ‘Example 19. Find the eigenvalues and eigenfuunctions of the operator d/ dx Eigenvalue equation: s d y(x) is the eigenfunction) 2 www.careérente vour.com Note: (1) If the state of the particle is an eigenfunction of the operator 4 , then the expectation value of the physical observable corresponding to 4 willbe equal to the eigenvalue of 4 corresponding to the state of the particle, (2) The state of the particle is given as: 244) Iv)=C)lA)+Calds) +6 |6,)+ are the eigenfunction of the operator 4, then expectation value of the the physical where |6,),/¢2). observable corresponding to A willbe thasinanvaliae nfomarnine 7 where 2 a1 Soin. Soln, Soln, Se = Concept of wave function Example 20: A particle of mass ‘m’ is confined in a 1-D box from x =~2L to x= 2. The wave function ax of the particle in this state is y (x)= vo ow 2) (Find the normalization factor. (@ The expectation value of Pin this state ro ot a(x at 2a: @ J vvae 1 = |v) Jo (Bet =>|vol [ 4[ cos? ad 2 Mo (2x) 4b Yo leah xin 28) AL)" at bl fer 20y] 1h 2a 2 cas? (2) (=) a 4L AL, Example 22, The wave ayooraReheasclekeHOLeReGettqmD. whee N=(/na)". Evaluate (Ax)(4p) Since y is symmetrical about x=0,(x) =0 ( Since, y is a real wave function, (p)=0 (ot) g? fox? (p*)= (tay? fool $=] Soe0[ SE jan ae id a 2f et exp| ldo w fe Ja a Example 23, The Hamiltonian operator of a system is H = -(d?/dx*)+x*. Show that Nx exp(—x? /2) is an eigenfunction of H and determine the eigenvalue. Also evaluate N by normalization of the function. y= Nxexp(-x? /2),N being a constant Co) a 2 2 2 eo, x 5 #)d faa) x Hys|-aree eee (5) =x oro) So) «o(-=)] Example 24, Con '), where A is a real constant. (i) Find the value of aac thi a unctions _ oromunin GARBER ENDEAVOUR AVOUR reerendeavour com . a Gi (v)= Jv" (4) | ono Jo (-Baikeo(—S Jena "ae Anyi) a? | Joo dx Concept of wave function . maa ate LE) [OOF] Ue s | =cin(-3) ote In the first term, the integrand is odd and the integral is from ~20 to oo. Hence the integral vanishes. Therefore, (p)=fk, since 4? Joo “art Concept of wave function Example 25. The normalized wave function of a particle is (x) = exp(iax—ibt), where A, a and b are constants. Evaluate the uncertainty in its momentum. if af = = Son, (p)=-infy eu = hafy tude = ha (p°)= rye Sa = (ia) [ytyde= fa? Uncertainty in mometum (Ap) = o*)-(oy" =Vha'—ha <0 Example 26: A particle of mass ‘m’ is subjected to the potential enetgy 7 (x)=—fx?. Ata particular time, it has the wave finction y (x)= Are™*”/@".. Ifthe particle has @ defnite total mechanical energy, find the value of ‘a’, Soln. The operator correspéhdifig a +(x). Lethe particle has definite viaue ofthe totgl technical ener Be si elzenfimction of H, that is, 3 P| Pie L gyadgeatiat eel a ad +g kte Bat ele? { 24h? 1 ma? mat 2 iat If Hy (x) hasto have the same fictional formas y (x), one should not have the x3e"*''*" term. So, 4 2401 (si mk 12 are ular Concept of wave function Example 27, A particle moves in a linear potential V(x) =~be, (.e, under a constant force condition), Show that (x) follows rigorously the same time evolution as given by Newton's laws. Soln, The equation for (x) is 2.5 Schrodinge! Consider a particle time, The total ener Now, the -D time-depe io (leayour.com ore. sar mine - tg pies ag. E (constant) m ‘Time dependent part: Ler E i ine = =Sare—L ed ot in h ae =T(t)=Ce® ‘Therefore, the time dependent part of the solution of the 1-D Schrodinger equation is independent of Mx). Concept of wave function Soin. Time independent part: Therefore, the time independent part ofthe solution ofthe 1-D schrodinger equation depends on thenature of V(x). So, the solution of the 1-D time dependent Schrodinger equation canbe written as v(xt)=X(a)T()= x (xe Now, if v4 (x5), ¥2 (xst)...47, (2,4) ate solutions of 1-D time dependent Schrodinger equation corre £ sponding to energies E,, Ey, Es,.--.B,, then the general solution is (xt) = (254) +¥2 (x58) + eta Example 28: The wave fi i i Depolehtialis y (x)= Aexp(-a°x* corresponding to eigen @ 26, (1-a2x’) 9) x ¢ ¢ Ei (1+207x*) Given: v (x) = Aexp| 2 > EY exp -< Replacing all these the FEDER SaTEROW) a Memos ( 2 WW: ree ANOUT.C | = = Aexp(-a*# Lgpreerendeay V(x) + Aexy )= EAexp(-a7x") n # Ba? _ na? = ~jylte- 2a +V(x)=E = V(x) Spi ttt + = te | Therefore, V(x) =2% + Fe 2kyats? | Example 29: The wave function of particle moving in a 1-D potential V(x) is w(x) = exp(~iax +b). The potential V(x) willbe the form: @V(x)ex ()V(x)ox ©V(x)=0 @V(x)ne™ i The Concept of wave function a Soln. Soln, Ee ry Ey ake +V(x)=E => V(x)=0 Correct option is (c). ‘Example 30: The ground state (a part from normalization) of particle moving in a 1-D potential given by, x o(-S]oos (V2x). The potential V(x) is taking m=1, 2 =1) @= (0) -VBtani(VBx) () $-VEtan( VE) @%-cot( Bs} sinh (2x) a LCABERENDRANDUR | aaah soso sana ANN > -El -1--yvmw dabeerendeavour.com Since, energy ofthe particle is independent of, then > r-E(e ~2Nitanh(V2x)}=0 = V(x)= Ee ~2v3 tanh(v2x)) Taking m=h=1 weget, => V(x) 7 v2 tanh (V2) ‘Superposition of States: If|¢,),|9:).|4,)-.-¢{@,) are orthonormal state vectors/wave functions and are representing state of quantum ‘mechanics system, then any linear combination will also represent state of a quantum mechanics system, in other words if 4, mB, are solutions of Schrodinger equation, then linear combinationsuperposition of thestates Le. Concept of wave function Soln. Soin. |v)=C|4)+C.]4)+C,|6) +. willalso be a solution of Schrodinger equation. Note: _ (1) |v) will be normalized if |¢,/? +|C,/? +|c3| (2) Probability of finding a particle in |¢,) state = Example 31: Consider a system whose initial state at t= 0 is given in terms of 3 orthonormal state vectors |) I4s)s16) as 1 1 |v (20)) = F51A)+ 4|és)+ 7516) (Aisareal constant) (® Find the value of so that |y (x,0)) is normalized. in|), 4) 4) states. (@ What are probabilities regarding the system to be (Gi Find the state of the Site Example 32: The wavdhhbAW PALRRKARA BAY PULADM) where |) and |) arethe | normalized eigenstates with energies E, and E, respectively. The hamiltonian in the state y is E, ~2E, 2£, @ Be, Be, et oS 1 Suppose normalized wavefunction: |yy) = ¢[ 8 \#)+18) (where C is normalization constnat) 2 2 Normalizationcondition: (y|y)=1 => Chel acap ‘Therefore, the normalized wavefunction will be Ww)=-bia Flay tion vethe Concept of wave function Sotn. Energy ofthe particle inthe given state willbe (E)= P(t) Eo +P(A)E 2.6 Various Representation of wave function: ‘Wave fianction in position space is represented by y (x) ‘Wave function in momentum space or k-space is (p) = tal fv@ le _ Inverse relation: y (x) Ifa wave function is normalized in position space, it will be normalized in momentum space. But the normal- ization constant may be different. Example 33. Find the wave function in kspace for the wave packet, fa. Example 34, Find uMMAMGARRAKARdeAVOUT GORE v(x )= Aer ea 2082) Fimste-n ‘Therefore, the momentum space wave function willbe 6(p) = AV2z1h 6 ( py ~ p) Example 35. Find the momentum space wave funetion corresponding to the position space wave function ofaparticle at a given instant y (x)= Acos x $(p)= J doostee ae at Pa ¢ Soln. Soln, q Concept of wave function naff Bade Ha Sy dah[ 3 (nk p)+8(nk+ p)] =A Vala (nk p) +6 (nk +) + Probability Current densi As the time changes from t=0, the probability of finding the particle in some region of space rrayincrease or decrease. Ifthe probability increases in some region, then it should decrease in some other regionsuch that total probability of finding the particle in the entire space should be equal to one. We can assume this as a flow of probability from one region to another region, like a fluid or current, Therefore, the probability flow satisfies the equation ofcontinuty ie. VI+ ee 0 ot where p = probability density =y"y and J = probabil ‘The magnitude of pro : i s i.e number of particks passing through per unit area t is along the direction of flow of the particles. , (f= wave number, Bxample37, on MMNaLeSTENGeAIQUEGDI. (A) The probability current density for the real part ofthe wave fimnction y te ak @) OF Om @o (B) The energy of the particle is 42 242 22 2g @ Shtk 6) 3h°k ak ak om 2 a (A) The real part of the wave function =e +2 is 3cos kx ‘Therefore, the corresponding probability density willbe zero Correct answer is (d) a a wid Ps ix) | B 4 ket —2iK eo ® fy ( Sma et et 21K € i Concept of wave function Soln. Soln. Correct answer is (c) Example 38. Calculate the probability current density for a Gaussian wave pocket care “Ribnchison Example 40, Show that for any normalized wave packet in one dimesion { I(x)de=(p,)/m, J 4(@)ae “tive Lyas— dvs iva -afIralagper| where c.c. is the complex conjugate of the first term. Now.(p) must be real, So, the first term in the square 1 1 bracket is — (Px) and hence its complex conjugate is =;(Px). Concept of wave functioiconc, [Flo] Example 41. The solution of the Schrodinger equation for a free particle of mass m in one dimension i wm [ewes ie WOt).At = 0, y(x,0) = Aexp| =) Find the probability amplitude in momentum space at j= ane \ at time t, Soln, 2(7.0)= ee | venpe- Bae Q3. = AT exp PE Aa of Pia Vink jeo( ae as Noh on 4h Soln. Qt (@) 2ih xp Soln. [x,p?] eee “(aap Correct Optionis (). Q.2. Which ofthe following isa seladjoint operator inthe spherical polar coordinate system (r,0,9)? [CSIR June20121 gg ino a ino a oe eh ee ihsi Fin 6 00 O- © 36 (sin Og Soln, Anoperatoris said to be self-adjoint if [4° Aw de= [(44) yde v {i [60.0 eS |v (nays sade oa =~inf i fe (1,09) )(1,8,9)r2drdo dp 06-060 inctio! Concept of wave function rension i, £=0 and Q3. Soin. Q4 On integrating by parts w.rt @ weget, = [ j 5G 2 ay wr? sin0drdadg sind 6 yr sinddrdddo : oye sinBard0dg = Correct answeris (©). Given the usual canonical commutation relations, the commutator [A,B} of A=i(xpy~yp,) anc B=(yp, +2p, is: [CSIR Dee 2012] (@) A(xp.—Pxz) ©) “A(P.-Py2) ©) A(x. +7) @) “ACH, +P,2) A=i(xp, ~ (.B]=[i(on, yp) 8. =i[xp,. 9p eilyn.. u au y Therefore, ti ei eet com Correct option is (c) Ifa particle is represented by the nostnalized wave function WBlt—»!) — for -a0 (b) @(x) = Boos x,B real (©) @(x)=Cexp(-D/x*),C>0,D<0 — (d) @(x)=E xexp(-Fx’),B,F > 0 Concept of wave functio, 14, Anelectron ina time independent potential is ina state which is a superposition ofthe ground state aa (4-11 )and the st essed ae (£, = 7) The wave faction ofthe destonwilrepeat elt | with aperiod of (GATE2003, | (a) 3.11078 seo (b) 21x10" sec (c) 1.2107! sec (d) 1.010 see ' 16. Apatticle has the wave function y (x,t) = A[ exp (iar) cos(kx) ]. Which one ofthe following is correct? | I (GATE2003] (@) This is an eigen state of both energy and momentum, 24. (b) This is an eigen state of momentumand not energy (©) This is an eigen state of energy and not momentum, (@ This is not an eigen state ofenergy or momentum, ‘ Commom data for Q. 17 and Q.18 Ys. The normalized wave function y, and y, correspond to the ground state and the first excited state of patil ina potential. Youateivet the wave finctionsas Aiy, =, and Ay, =y, [GATE 2003] i 17, Theexpectation value he, (@)-0.32 ' 18. Which ofthe following (a) y, and y, i) heither y, and y, 19. Thetime independent hservation ofthe i [GATE 2004] (2) total binding energy Hef the system, i (©) total kinetic energy off | yn 20, Asystemina normalized site| AR BER of the system, requires that the constants’, aiid ¢; ust satisfy the COhidition www.careerendeavour.com IGATE 2005] @elel-t — Wlelref=t © (elles Ol 1 21. The parties of the wave functions (i) coskx and (il) tanh kx are IGATE2005} (a) odd, odd (b) even,even (©) odd, even (d) even, odd . 22, Ifthe probability that x lies between x and x+dx is p(x) dx = ae, where 0-< x <0o, a> 0, then the probability that x lies between x, and x(x; > x,) is {Gare 20 (EME) alone) WEN (eM —e™) MERE -e™) 23, The wavefimetion of particle ina one-dimensional potential at time t = 0 is w(t=0)= Fel 2vs(x)-wi()] unctlo concept of wave function ) Citself 003) | 125. state off, Ayrey, 003] | 26. ' | 004) lan. sigenstates 005) 05) 28. the rye ee where w, (x) and y, (x) are the ground and the first excited states of the particle with corresponding energies, E,: and E,.The wavefunction of the particle at a time ‘t’ is:. [GATE 2006] : oe ) eo 2el)-W 0] ) Jee [2¥0()-wi()] 1 : © xe vals-vi 00] (@ FgLvolder*—wilsje"*] A particle of mass m is represented by the wavefunction y (x)= Ae™ , where k is the wavevector andA is constant. The magnitude of the probability current density ofthe particle is IGATE 2006] a fk a (Ak) (nky @lAry Alon ape y @ ap ae ‘The wave function ofa particle, moving in a one-dimensional time-independent a) VC»), is givenby w(x) =e", where a and bare constants. This ‘means that the potential V(x) is ofthe form. [GATE 2007] (a) V(x)ex V(x)oce™ A particle is in the ergy eigenstates |E, =10 eV) and |E, =30eV). Theg is 20 eV. Thestate |y) isgiven by [GATE 2009] tate? N isa constant and o.,>0. {GATE 2009] © yenewte® ‘careerendaalgtco™ trek ‘Common data for Q. 28 and Q. 29 ‘The wavefunction ofa particle moving in free space is given by, y =e" +20" ‘The energy ofthe particle is [GATE 2012] a 7K snk? a as @ on ox one ‘The probity cent density rte tel prof he waefinetn is ak fk, @1 OF O tn @o 30. aL. 32 33, 34, Concept of wave functionicone Consider the wave function 4e!(r, /'), where 4 is the normalization constant. For r= 27), the magni 35, of probability current density up to two decimal places, in units of (47Ak/m), 18 _. | (GATE 2013) | Ina quantum mechanical system, an observable is represented by an operator 4. If |y) is a state of the system, but not an eigenstate of 4, then the quantity | 5 36, r=(vldly)-(u|#|v) satisfies the relation (TIER 2013] @ r<0 ) r=0 © r>0 @ r20 A particle of energy £ moves in one dimension under the influence of a potentials V(x). If F > V(x) for, some range of x, which ofthe following graphs can represent a bound state wave function of the particle!, (TIFR 2013} ‘37, @ © me vores SE AREER ENOEAVOUR) where 4 isareal Theexpectation velue of its momentunr [TIFR 2014] wt tel #) wun. pareerendegyaur.com., Atwo ste quantum system has two observables 4 and B. It is known that the observable A has eigenstates |q,) and |q,) with eigenvalues a, anda, respectively, while B has eigenstates |f,)and|8,) with eigenvalues b, and 6, respectively, and that these eigenstates are related by (TIFR 2013] |A)=Zla)=4}a).—(fr)=4la,)+ Zl) Suppose a measurement is made of the observable 4 and a value a, is ae Ifthe observable B is now measured, the probability ofobtainng the value b, will be (a) 0.80 (b) 0.64 (c) 0.60 (4) 0.36 inctlon concept of wave function agnituds.35, t 5 ma] | a state of 36. B) | Vex) for, » particle 13] 14] genstates 1») with 15) Bisnow B ‘The wave function of free particle in one dimension is givenby y (x) = Asin x+ Bsin3x. Then y(x)isan eigenstate of (JEST 2012] (2) the position operator (b) the Hamiltonian (©) the momentum operator (@) the parity operator The quantum state sin x|t)+exp(id)oos x|), where (t]|¥)=0and x,¢ are, real, is orthogonal to: [JEST 2012] in x{t) (b) cos x|t)+exp(id)sin.x| +) (©) ~cos x] t)-exp(i9)sin x|4) (@) ~exp(ié)c0s.x|1) +sin x|1) Ifthe expectation value of the momentum is (p) for the wavefunction y(x), then the expectation value of momentum forthe wavefunction e™*y (x) is {SEST 2013) @k () ‘The Hamiltonian o tates corresponding tothe larger i [JEST 2014) (v2 +1}|2) 3 +1))1)+|2) € g is a constant) and respective @ |) -(2+1)l2 © |1)+(v2-1))2) Consider a three-state ay. eigenstates "Hthesysemis alvin Déeneerendeavour.com wi=|o ° What isthe probability that ata later time "system willbe in state 0 \vs)=}o (JEST 2014] L 3gt gt E-3gt) @o 5 Asin’ (#) oF feos 38) ( dsin ( ie ] Concept of wave functior, “Aball bounces back off earth, You are asked to solve this quantum mechanically assuming the earth ia infinitely hard sphere. Consider surface of earth asthe origin implying V (0) =o anda linear potential eleswher, (ie. V (x)= -mgx for x > 0), Which of the following wave function is physically admissible for this problen, (with k> 0): (JEST 2014] = 2 @ y (x)= Aze™ @ v(x) ke = Ovex Ov( Axe! AREER END! Se RHE www.careerendeavour.com ‘unetion earth is af One-dimensional potentials 3.1 Introduction to Bound States : | Ifthe motion of particle is confined to a limited region of space by potential energy so that theparticle can ' move beck and forth in the region, then the particle isin abound state. The simplest example of ll motions in bound state is the motion of a particle in a one-dimensional box with zero potential energy and the potential energy is assumed to be infinite atthe walls ofthe box. ‘The motion of a paytioleitt ig x oscillator, motion of at x, motion of a simple harmonic ith (E < Y), motion of a particle cle in one-dimensional (also in potential, ete. are the common equal to such problems leads to of discrete energy and non-degen- (i) The wavefunct ric potential should have adefnite Parity ie. it will (i) The wavefuncti responds to the ground state of the particle and Wall ‘ state of the particles. 3.2 One-dimensiofip ilfely ech Phat Wal Consider a particle of mass ‘m’.and.e ving along X-axis, inthe region fromx = 0 tox=a under *seswtemenivw.garegrendeavour.com 2% otherwise Veo Vaw ve v=0 x50 xéa — Figure : Schematic diagram of 1-D infinite potential well, ‘The particle experiences no force within the potential well but feels a sudden large force directedtowards the origin a it reaches the points x = 0, a. Since, the potential is infinite outside, the particle cannot penetrate outside of the region 0.< x &, =. 242 Firstexcitedstate energy: n=2. => E, = 27 mal Ingeneral, E, =, Difference between} Spacing between t particle ‘m’.As a $4 GEE aotrhing a continuum of energy. PAREER ENDEAMOU z as = witere's* vac aummutateanantleaveus coms.” Difference of momentum between two consecutive energy levels fe My Ronal Eten ont ga a a a Various eigenstates of the particle inside a 1-D box: i vie) a en Ground state First excited state One Dimensional Fotentiaj One t | i | Note: y, (x) has (n-1) nodes (excluding the boundary atx = 0 and.x = a) ‘Variation of probability density corresponding to various eigenstates of the particle: Probability density of finding the particle in the n® quantum state is given by, 2, (2) =n (f= Zsa? i wet " he et VN ier VN - | t Figure : Variation oy ‘density corresponding to ground, first excited and second excited state respectively Note: Stinging the particle inthe eta i: | ' orthonormality condition 4 i: i : : (x) (whichis well behaved and ‘igenfunctions as 4 i i Soln, « ( (=fv(-02) yde=—(-ih) 2am fs) 7 cos (Pe)ar-0 oe Note: Expectation value of momentum for a real wave functionis always zero, (#)= Ive yar = ae sin? de eee ireer eal qe (psindnnx/@ 2Qnala Example 1: apes ina aS ATS ‘extending from SF w.careerendeavour.com v(x lla) ) +2|4>) | whete |) and |p) denote the ground state and the first excited state wave functions. Find the expectation of x” in this state. sam. (*)=(vlely)=—fe((Al 20h) Fella) +214) = ELIA) *2(6 ble) +2 (dala) +4(62Iol¢)] One Dimensional Potential Soln, Soln, Soln, 32h gL) 6ab ont 2) 2 45a? ue 32L Therefore, (*)= 5] 5 ~ 9 Example 2: Calculate (p,) for ¥ (*) alla) where |g) and |) are the ground state and the first excited state wave functions of a particle in a deep square well potential. 4 ein 27E oye EH J2sin=* cos de aL \f “x” in this state, The 25 nD Example4: Apartice fs x= ofthe wellis shifted to’ 292 rT. yi range 0 Psin( ome ak sin 2) = v(s0)=Ala)+ ea ele) Normalization condition: (y(x,0)|y(x.0))=1 => > |Gf=1 hts PS oae lf 5 Ve One Dimeneional Potential fe, E fs fc 0)) = J 4d) + Js) + Therefore, |w(x.0))= ysl) yeQ lO) vig lt) (b) The state of the particle is a linear combination of ground state |¢,) , 2nd excited state |9,) and 4th excited state |@,) respectively. Therefore, the energies we will get are ont 2ma?”™? 2ma* ‘Now, the respective probabilties are PE )=KaiwiP=2 PE)=leiwPati PE (£) = P(E;)-B, + P(Es).£, + P(Es)-Es MP3 Ont Sah t ~3°2ma* "10 2ma® "10 2ma? we “(3 27.5} 2908 2ma°L" 10°21" 10ma? © |v(=4) EA th i ) bean)eneeh = P=oeniv)l= 0 Example 7; A particle of mass mr-confined to move in a'potential V(x) =0 for 0 me Y Vays = Ey; > 2m(V,~E. : Aasuming «= = 2m(o-B) and B? = 2 Region-1; an -ay, =0 Region-2: Me + By, (Sb) and (Sc) can be written as (5b) (Se) (6a) I | | | | i | | | | (6) 19 ma, =0to (6a) (6s) One Dimensional Potential ) Region-3: 43 -a’y, = 6) Solution: ae (68), (6b) and (6c) willbe ofthe following form S =A ax ae Vi (x)= Ae + Be ssf } ae Wy (x) = Ccos Bx+D sin Bx . 3 ( wlx)=Fee see Boundary conditions: - 2) @y (x) > 0atxoa0 “ v(x) ona Hae continuous at x= Using the fst condition, we get B=0 and G=0. Since the particle is moving under a symmetric potential, then the wavefunction of the particle should have a definite parity i. eth (2) Even bound staf ‘The wave function df Now, boundary con ® 0) ovstgeqanec categiendeavour. com a We know that, asp ue a Defining BS pandas =g,weget pram p = q and p? +g? = 03) Ira ‘The second equation represents a circle in the p-q plane with radius R = (ae which depends upon the the height M, and width « ofthe finite potential well. Both of these canbe solved graphically and the points of 3 mnt eee One Dimensional Potential Pr = Pos P2»Par- which also gives the discrote spectrum ofenergy valu for the even eigenstates as (1 =0,2,4 6.00000) pager’ ‘Number of even bound states: 0 (w#1)even (2) Odd bound states: oe ‘The wave function of the particle will be of the form: ~ wila)= er" Now, boundary conditi «sy ihe dS vt | Wl re) dy a "5 ADeosp 2 Malye 28) Wal p22) ae? =pDcosp* (16) | fib cated ondedveur con > i i Dividing equation (10) by (9), we get, pox BS =-a an | i 2mV, We know that, op aae 18) Defining pS=p and a q,we get | ha? poot p= —qand p? +g? = Meet! (19) 2h? Both of these can be solved graphically and the points of intersection of the curves] 2 gand p+ 4 ives different p values ie. nia’ He (1s) (16) ay ca} yy curve One Dimensional Potential ‘hich also gives the discrete spectrum of energy values forthe even eigenstales as apt PAPE (1 =1,3,5.Tonens) ma’ eotp=4 ‘Number of Odd bound states: is <= = one odd bound state Example 9: There are only 3 bound states for a particle of nmass ‘m’ ina 1-D potential well of the form: (x20 = S| One Dimensional Potential Soin, Soln, Sol. The depth of the potential wel satisfies the condition: anh? ont 2a ann S othe first branch does not exists. therefore, ee ee pe pe TET] 7] mE tial atial of | seurve na/2,} des not | One Dimensional Potential = Soln. Soln, Example 12, Consider a particle in a potential shown in figure below. Write down the wave unctions of the particle (having energy eigenvalue E) in the different regions for bound states. Asis clear from the figure, the energy E should be negative for bound states. We can define four regions. Region-I: x<0, Region-I: 0<2x< Lj, Reglon-Ill; L, L,, The wave function fora bound state at energy E can be written as, (solving 1-D time independent Schrodinger equation) 4 72 ERB a Inthe first excited state, the wave function is ofthe form y-=-Asind . The maximum value of the wave tnctonisa coll GARRERGILCRY OL FROIN, (kL kk an Aw Asin eee Asin e773 Pe wos 44) A si Bd “+ 2L 2 L . The wave function at x= 5 is y= Asin 2n Example 14.A finite square well potential has length Land depth V, A particle of ass 5, 7 is trapped in ‘ this potential. Show that it has only one bound state and the energy corresponding to this states satisfies cos((ET%)= EM One Dimensional Potential 2 LY [2mr, Soin, p> +a (3) [20 an VoL? | Moe wy on [a Thus, VP” +4? <5 and hence only one bound state ocout. Using, p= BE mine rd woof 3.4 One-dimensional step potential: Let a particle of mass mand kinetic energy # is travelling Pen potential step at = 0, given as following: Via)= { 22 to the x-axis and is incident on the 0 forx<0 Vo fore 20 The energy E of the — From classical point of vie will be reflected back-at be reflected back at x =O aste Quantum mechariclly wil be shown that th-letronewillbe pry ¥feted and party transite at X= 0.This so TAR ACEH EET EN GEA VOUT.COM Case A: (E> ¥,) ‘The time-independent Schrodinger equation for We dy, dy, ~p oe i Region 1: ~5 T= Ey, = ah (20) Pd Py 2 Region 2: Soe Ey, » Ee Voy =0 v2=0 (21) mls 2m(E~V, where, @ = ie and B= eae ‘The general solutions of equations ate atax «mix >0. ‘cle not dat 20) (21) an | | One Dimensional Potential ‘The terms e!* and iar represent respectively a wave travelling along the positive x-direction ie. incident wave and a wave travelling along the negative x-direction, ie., a reflected wave. Again, the terms ¢* and eB sepresent respectively a wave travelling along the positive x-directis ‘wave travelling along the negative x-direction (a reflected wave). But due to the presence of only one discontinuity at x = 0, there cannot be any reflected waves in region 11. Hence D = 0. Therefore, Applying the following boundary conditions i.e transmitted wave and a (dy, dy; © Weo=Wano ad Cid (2), . wewillge, B=222.4 and c={ 2% la B a+B therefore, wavefuunction in region 1 and region 2 will be (23) oe 1p Tava Gaeorit caveat CONF Jytde= a4? Cee =p, sap f= a we I td,4, “+p (ab) ie. total number of particles is conserved in the process. Reflection and Transmission Coefficients: Reflection coefficient ay _ Reflected flux (J,.) lah. (o-8 ie vE- i VE+\E-% Incident flux(J;) he ge lal m™ One Dimensional Potential “Transmission coefficient Transmitted flux (J,) Incident fiux (J) 4JE(E=W) (eer) (a+py Note: Aap i) T+R= ON eB) (il) For E=V%,R=1and E>>V,R=0 (iii) For £=%, 7 =Oand E >> V4, 7 = (iv) Amplitude (C) of the transmitted wave is greater than that of the incident wave (4) as ct > B. (v) De Broglie wavelength in region I will be shorter than that in region I. This is due to the fact that the kinetic energy of the particle agate is greater than the Kinetic IL. The nature of the wave forms is shown in figure, ~ am The analysis thus sho certain probability of Case Bt 1 <¥) a = (2m@o-£) where, a. ae and B= ro The general solutions of equations are = All 4 Be, yy = Ce 4 Deb 26) But the term, e* ~+ 00 a5 x~y00 which contradicts the condition of well-beltaved wave function, Therefore, C= 0. al de te One Dimensional Potential Applying the following boundary conditions: . (ay \ O Deo = deen amd DE) ae hg we will get, B= iB ana po( 2 \4 ia—B fap. therefore, wavefunction in region | and region 2 will be ie y= alo" (SB ior io (27) x0 a ‘The phenomenon is very much similar to the case of total internal reflection of eleetromagnetic waves while travelling from denser to a rarer medium. Quantam mechanically, the wave function of the particle in the region 2 is non-zero. therefore, the probability of finding the particle in classically forbidden region (region 2) is P(x) =|va(w)f =[ph ‘Such penetration of a barrier by a particle (having energy lower than barrier height), falls exponectially with distance from the step Dotential barrier. Soln, One Dimensional Potential Penetration depth: It is defined as the distance at which wave function reduces to I/e of the value of the wave function a the barrier (x = 0) and given by 5 ate ‘The fact that particle can penetrate into the classically forbidden region leads to Quantum Mechanical ‘Tunnelling. Ifthe potential barrier is infinitely high Le. Vy — oo, then penetration depth becomes very much negligible Example 15. Electrons of energy 3.8 eV are subjected to.a potemial = 0 for x <0 and = 7.6 eV foc x> 0. (a) Find the wavelength of the sinusoidal vatation of the Wave fumction in the classically alowed region (b) the reflected wave differs from the incident wave by a phase factor. Find this phase factor. WW. Gareerendwavour.cormn, a a < |—— 10am 38x10 e7 nm’ (a) The wave number *k? The wavelengthis 4 = 2% = 628 a nn (©) The wave function in the classically allowed region is = 0,63 nm a4 ear, 1atB iar vale tbe] Thus the reflected wave differs from the incident wave by a phase factor Webave a =10.nm"' and similarly B the sical uch, One Dimensional Potential Soln. Soln. Example 16, Electrons of energy 10 eV are incident ona potential step of height 13.8 €V. Find the distance in which the probability density of finding the particle decreases to a factor of 0.01 as it penetrates into the classically forbidden region. ‘The wave function in the classically forbidden region is ofthe form ce According to the question, P(x+d)=0.01P(x)=|cP e7""4 =0.01|C/ & => 274 =100=3 2yd = In(100) => d = 212 = 0.23 nm Y Example 17, An electron beamis incident on a potential barrier of height 0.03 eV and ofinfinite width. Find the reflection coefficient and transmission coefficient at the barrier ifthe energy of the incidentelectron beam be (@) 0.04 eV, (b) 0.025 eV. (a) In this case, E > Vo, Reflection coefficient (ee ‘Transmission coefficient = (b) In this case, E < Vy. reflected back and no ¢lectrot R=1-0.1=0.9 T=Oand R*1,,i¢. all the electrons that are incident on the barrier are ‘will be transmitted through the barrier. 3.5 One-Dimensional Rectangular Potential Barrier Consider asveam of particles each fis sm’ and energy E be incidént from let on the potential barrier surface at x = 0; given as Ho The following two cases arise: Case 1: If E> V,, then according to classical mechanics the particles will be wholly transmitted and no reflection is possible, But quantum mechanically there is always some probability of reflection at x= 0, back in region I. Case 2: IfE < Vy then classically the particles will be wholly reflected and hence penetration through the barrier is impossible, But quantum mechanically there is always some probability of penetration into the barrier and appearance of the patticks in region IIL. This finite probability of transmission through the barrier even EV, is called the quantum mechanical tunnelling effect. One Dimensional Potential Case A: When E > V, ‘The time-independent Schrodinger equation for dy ayy, 2mE PU ory, = Region 1: ee My => Pe Tra =O => qe te 70 08) as aya dy, 2m dawns g Region 2: —~— aia +H = Ew, => z +7 EVov2 =0 => “gi th ven0 (29) Pays yy mE Py, Region 3: - oe =EV; => ae > W3=0 = Br tewsno G0) ImE |, 43 _ 2m(E-Vo) where, a? =" ang p? = Eto) : e e ‘The general solutions of equations are ES + Geri Since there i no refi Applying the allowing, G1 went cof NG BE ROE es Substituting the “ac WP G@NREIENNISAVOUECOM rs ee[-ta 4] 2 “[0- ee a ee foe-gie| eh A= From equation (33), bial 28) 29) 30) sy i4) One Dimensional Potential ee 5) 4 @s By(eF*—e) -2ap(eP +e) 2aPeosfa— ia? +P)sinpa Reflection and ‘Transmission Coefficients: Reflection coefficient hae Reflectedtux(J,) _g !2P_ ype p= Reflected fu (J) _ m1 BI 1 a oe 4E(E-V) foe, (il) As B increses ay ll sito rapid compared to sin? ea, for ro eeuentolcomn.. ef he pari in region M1, then there will be perfect transmission of the incident beam of particles. This is Ramsauer- Townsend Effect. (v) For E >> Vy, T= 1 and R=0. The natures of the wave fimetions y,,W> and y3 in region I, II and III respectively are shown in figure. Region Regiontt Rent “ampli q@ One Dimensional Potent Case B : When E < V, The time-independent Schrodinger equation for chk e will get, A= £1 we will get, <( i Applying the folowing! : @ Wa)rea= ) AREER, BM F (® we will get, C= = Bet C= a (ental Substituting the conc ORMGATRELERARAVQUECET set, aC meee F 2ape i A 2afcoshBa+i(B? — a)sinh Ba (40) tial G6) e7) G8) 9) (40) a) (42) One Dimensional Potential Reflection and Transmission Coefficients: Reflection coefficient _ Reflected ux (J) _ “in wise _ [BP Incident flux (J;) cp TTAP m 24.92)? (eA sinh? Ba 40°" =R= 2 7 w +B?) gy t+ sinh? H{ “t) sinh’ Ba 45%-E) be ) ‘Transmission coefficient Tease fx) 7 — a 4a" cost + ant ba 4EVo-E) Nate: ( There isa finite jal barrier of width a and height V, even if E < Vy, (ii) As E increases, ( | The rate of decrease of sin” Ba for a fixed value ofida fase steadily with E. i : i [reste iwwrcat6ereniieavour.com 16E(%-E) W sinh?(pa) Therefore, T= ze The natures of the wave functions y;,W2 and y3 in region I, II and II respectively are shown in figure. qe One Dimensional Potential Bev, E-V, E>V E=>v, ES Example 20: Calculate the percentage transmission of an electron beam of energy 3¢V when incident on a potential barrier of height 4eV and width 20A. E Soln.: ‘Transmission coefficient r= (- ze ‘ ‘What is the probability that t wil escape from r ? a tthe surface of the nucleus is 4 MeV and the mass} Soin, an ww career endeavour. com Given: E = 2MeV,V,=4 MeV,a=2x1.4 x10" ‘Therefore, transmission co-efficient will be T = 0.124 3.6 One-dimensional linear harmonic oscillator: Ifa system vibrates about an equilibrium position such that (a) the magnitude of the force acting on itis proportional to displacement from the equilibriam position, (b) the direction ofthe force is opposite to that of displacement, then tis called linear harmonic oscillator (LHO). A simple pendulum, a spring -block system, a floating body etc are familiar examples of a dynamical system that perform simple harmonic motion. In quantum mechanics, linear harmonic oscillator is used in place of simple harmonic oscillator. Classical Linear Harmonie Oscillator: Leta time ‘t, the displacement of particle of mass ‘m’ constrained to move along the x-axis be x= 0 fiom equilbirum position. Restoring force on the particle is j° = —fx (kis constant). aton y that | us is One Dimensional Potential a re ax eee = Sr arx=0 ? ne Get Oa 0 (where 0 ‘The solution of equation (1) can be written as x=asin(or +) This represent the simple harmonic motion (SHM) of the particle with frequency, 2x 2x ‘Therefore, potential energy of the particle £, and Kinetic energy of he particle apm Quantum Analysis, A particle of mass‘ aN EOI} fy ay ay 2m £4.28) p-Lne'zy 0 Co Solving equatian (42), we get the normalized wave function oflinear harmonic oscillator in the state is 2 a meta ame = H, (ax) = v, (x) (x r| é (ax) where, & "i (43) First few wave function of the LHO are as follows: ‘Ground state (n=0): One Dimensional Potential va First excited state (n=l): wbl=(s3e) (2ax)e**? Second excited state (n=2) vie) -(%5] (4x%a? -2)ere?? Third excited state (n=3): yaa) Note: (i) y,, (x) is anf Gi), () has Energy eigenvalue of 4° excited state 3 excited state 2" excited state 1" excited state Ground state On: (Solr tial Zal | o oimensional Poveneiat &S + Existance of ero point energy according to Uncertainity principle: Let a particle of mass ‘m’ executes SHM along x axis and at time ‘t’ its position is x from the mean position. Total energy of LHO E = E, +E, =2—+~. A Ax= x the = Sinoe, A=. then Ap = 57 => 2 Therefore, B= (#5 aita? Pg Problem 24: anf where y,and y, a expectation value of eng 31 7 (@) Sho iC to whw.careererttl y (x) should be zero at x =0 ‘The normalized wavefunction of a particle moving under a linear harmonic oscillator potential is v, (x)= Ae? 7H, (ax) [view na y(2) will be non-zero forn=0,2, 4, 6 wn. and will be zero fort=1,3,5,7, So, possible values of'n’ are odd n= 1, 3,5, 6, . () The energy eigenvalue of the particle willbe (Gi) The expectation value ofenergy will be (6) = PE, + PE = dS hos de dame am One Dimensional Potential Example 25: Ina 1-D harmonic Ostillator, two states are defined by = 26, +34 Va= hhh rae, where ¢,, 4,4, arenomulized wavefiunction corrsespondingh to ground state, first excited state, second stat respectively, (@) Thevahe of a forwhich y, is orthogonal to y, is (@2 1 @-1 (d)-2 (ii) The expectation value of the oscillator in the state y, is @ tho She @Sto @ tho Soin. é, Lia ‘Example 27: Apartic! potential V (x) = yh the spring constant is sud ticle in the groundstate of ney} potential Soln, Normalized ground aaj 20a" : = i Kivole £( 2 ae i a @avour.com fc ras (2)" 2 | meme" mo “Tad aor mA (1+v2) Operator formalism of harmonic Oscillator: Hamiltonian ofthe system is Defining two dimensionless operators paoh and X=$,/"" Ineo ntial One Dimensional Potential ad state » afta of new} The hamiltonian can be written as, HT = Introducing two non-hermitian operators, @=: plist, where @ and at are hermitian conjugate to each other. Now, aa =3[ £2 +P? +8 102] = ata fis As -(#art Sho >A= (w+) Dro where, N= atg = number operator/occupation number operatat, + Some important commu olaaj=1 fi ( [A,@a}=0 a i So, that, dn) is the eig & (Ea), Actionof @ ( on (2) gonerate ney rat (ol So, alm vi dareerendeayour.com Simi, 1 (4"|n)) = (414 + heat) =(E, +he)(at|n)) So, that, a" |i) arethe eigenstates of #7 operator corresponding eigenvalue (£, +h). Action of at on|n) generate new energy states that is higher by one unit energy Aw. at |n)=e,|n+1) > (n]a= (n+ lle, Therere, (nian) fey n+l] +1) = (n+1) fey? lel = VFL So, at |n) = VnFT|n41) ‘Therefore, the nth state can be written in terms of ground state as = invaltD ins One Dimensional Potential Calculation of normalized wave funetion: Ground state wavefunction: Since, action of @ on |7) generate new energy states that is lower by one unit energy fi, then alo) =0 fees [ma h 35 mea =0 = [me ay, +t ny, = V2 a eal ie Van * oman ”° = ayy <9 Ae te = me ong a oye Won svile)=aen[ ME) sy, (0)= Aon“) pew a? Normalization Condition: {5 (x) Since, at |0)=\1) veeromearoe cA 2h 2 ats? leg) cm-l] e ? A(ax) Expectation value of a odd operator for'a symmetric potential: ‘The wave fimetion ofa particle is moving under a symmetric potential V (~x) = V (x), has a definite parity ie it wil be ofeither even or odd parity. Therefore, expectation value of an odd operator = (4) = (y|4ly) =0 Similarly, 4") = (y| 4” |y) = 0 (nt = odd integer) Soln, al Soln, one Dimensional Potential @ Calculation of expectation value using operator formalism: gu lmes [re Asticknow, d= 8 + Pome ang a= therefore we get, tine) ns af PPA) = ohare Je) =e Lvaten- y+ VnrT(ninsiy]=0 le (y=} PS (pa A 1 teint) (nine) 7 = talnlla ea” vat +4'a)/n) Shing {m|n)+ Varn (n\n) Example 26: Let lola demree@crertckteres som to the ground and first state ‘ofa L-D harmonic oscillator. The uncertainty Ap inthe state a (|0)+11)) is: oak nih oe o (# (© dion @ Vinh (=A) (aha) (p= o+1 AE tA 20x (i) =AL (Ooh (oo) lplo)-Garh]+4[mind+0+0¢3mto] «mon ‘Uncertainity in the momentum in the given state will be dp=l\=(0 = Vimo One Dimensional Potentla, 242 morx? ata Example 27: The wave function of linear harmonic oscillator in the potential V (x) Yee at -For what value of will the function ay (x) be zero? given instant is veel Son, @ elt) = p(\Re* wis]. (RE 423) apa)" (Bx St ue V2 ap? a Vimo oe Example 28, Consider electric fie! E = Ey? .Apatt z & é 3 Snot onthe particle, Find the Soln. “aa ee 2k y: a | Rw. ae aici eavetitcom i ema? (-28) we | ma?) 2ma* ): the potential asa fimetion of x" is 262 This the potential ofa linear harmonic osilator with energy - “4 at x'=:0. This isa constant shiftint! mar | i 1) PES energy eigenvalues are (nok ha-t 4 So! Sol Sol tial —] one Dimensional Potential Example 29. linear harmionic oscillator isin ts ground state, Find the range of that isallowed classically, Calculate the probability of finding itin the classically forbidden region. Sola. Forclassically allowed region, the total energy E should be greater than or equalto the potentialenergy. For linear harmonic oscillator in ground state, this region is given by ‘The probability of finding the oscillator inthis range is Jig Wipe po {ne\,2 J joa) ae=2x ! fara —vhimao 7 ah ma Ae dt = 0.84 me, Assuming, t= es «the above expression becomes 2, Thus the probability of finding the oscillator in the classically forbidden regions I-0.84=0.16. ? thas wave function y (1x) at Soln, par. a acts at thy eas i BEER DAREER EN ; none tne pan REE econ igi al joffh Intwo time periods, itregains its original shape www.careerendeavour.com Example 31. A particle of mass ‘m’is incident on a rectangular potential barrier of height V, extended from 2,2 - i tox=L. The energy ofthe particles E =o +. Write the wave function inthe three tegions and mie by using the boundary conditions show that there is no reflected wave. Soln, ‘The general form of the wave function's th 4 pete = Ae 4 Be | w= f aon vir | ae a Ay 2" 4 Boe Js Win = 43 e where, &: aq One Dimensional Potential Boundary conditions at x=0, wrlx)=¥7(0) = At B= 4) +By dy,| _ dy; L u Sa) ea , Eh dé h 7 NAAR e(-A) oi) ‘Boundary conditions at x=, valL)=war(L) => Aye + Bye Ha! = Agel i) ayn) _ dv hol _p erital) — ip 4 eth eae = ky (Agel — yea!) = hy Ag’ (i) ‘According to the question, git2l . gi 0 where 8(x) is the Dirac delta function. Find the energy of the system. Sols. ‘The Schrodinger equation for such a potential is _# Py) 2m dx’ 7 V8) W(x) = E(x) ntial lensity pees Se Since the potemial is attractive, then £ <0 for bound states ao For all points except at x= 0, the equation becomes Soloution of the above equation, will be eae) wor fer x20 Q (the solution must vanish at x -> +0) The normalization factor is assumed to be unity. Integrating equation (1) from — to +2, being an arbitrarily small positive number, we get Example 35. Cons stant k. The interaction potential is H = Ayx, wher § Obtain the energy eigenvalues. Soln. The Hamiltonian veil Sat mot tyes fone 2m av: Hence the system can be regarded as two independent hatmonic oscillators having coordinates y, and y,. ‘The energy levels are Soln. Soln, CSIR PREVIOUS YEAR QUESTIONS 1 The wavefunction ofa particle is givenby V -(e +) tee Go and ¢, arethe normalized eigenfictions ‘with energies E, and £, corresponding to the ground state and first excited state, respectively. The expectation value of the Hamiltonian in the state y is: {CSIR June -2011) Ey Ey : Eo~2E, Ey +2E, @ yr pF @o“5> ary Normalization of y : w)=4l ia) a)] 2 Applying normalization conditionie. (y Iv) =1 ots a1 Correct option is (d) | A particle is confined ly a5 u(x) =tugx . The mean position ofthe particle R June-2011] k,T ay According to virial Th (Potential energy) For given potential u(x) HS INET BEL) = aia SHE) ccordngoomuinans ff W.GaTeeTendavour.com kyT => (KE) +(P-E. (Energy) = Sgt uy (x)=kyT => (x) Correct option is (a) Consider a praticke ina one dimensional potential that satisfies V(x) = V(—x). Let |yo) and | y,) denote the ground and the first excited states, respectively, and let | y) = 01g | vp) +0, | y,) bea normalized state with 1, andi, being real constants. The expectation value (x) ofthe position operator xin the state |y) is given by [CSIR Dee 2011] Sc ial mms jon ny note state ly) i) One Dimensional Potential soln. (a) 5 ¢Wo 1X] yo) tary, [x1 Y,) (b) 0,04, [yo X19, 0, 1X} Yo)] (©) aj tor (@) arya, (y= WHELY) (yas +(vals)x(aelys)+04/¥)) (ly) = 05 (Wo lx{ Wo) +03 (ys|x] vr) + toch (Yo [xl yi) + 04 (wi ls}¥0) Fora particle moving under a symmetric potential, the wavefumetion will have definte parity Le. ether even or odd, Therefore, the frst and second term will be zero as the corresponding integral will be odd innature. So, (&) Saf (volal¥:) +(wib¥6)] ‘Correet option is (b) Let | 0) and |1) denote the normalized eigenstates corresponding to the ground and first excited state ofa is ICSIR Dee 2011} = ele +a -2a'a-1] a | lar le |1) + (a (=O ~2(01ata) +(1faalt) + (ofan) +-(ala*afo) (010) +(112))-+(o11)+(110)) 0) + (ola"]1)+ (1 lo) + (171) + (ofa 0) +(y 1) +(0 ) ame) POM |==Ame = Ap = Vimo Correct answer is (c). One Dimensional Potential Soln, Soln. ‘The wavefunction of a particle at t= 0 is given by |y (0)) = glu) 4+fug)), where [u,) and |t2) are the normalized eigenstates with eigenvalues £, and E, respectively, (£, > E,). The shortest time after which |v (2)) will become orthogonal to |y(0)) is [CSIR Dee 2011] hi V2hn 2he ® EB © @-B) -E) (8 te srlet te? 2 _ (Bk ‘Therefore, the minimurp| se cil Correct answer is (b} A particle in one-di apaoneanagesps If there is atleast one bE AARON CL LIMES!) [CSIR June 2012} © wre Re 20? wre a ame Orn One Orne ~ wo if x<0 Oo; @ Vy if Oe Fd ay v0 V(x * Region I: y; 7 21 Region II: —“# Bes Vo)Wu2O => Ma +k, =0, where ki =e Vv) ix dx? 19 I So one Dimensional Potential iS tial Solution: yy =Asink,x+Beosk,x are Pq, 2mE &y, 2 _—2mE iM - iB igy = oki = mE a Region IM: Ga+ Wg =0= SE — Kiva =0, where Ki = Soln, Solution: yy =Ce** +De“** Since, the wavefunction should be finite everywhere in space, terefore C=O and yy, = De“** Boundary conditions: () Wi hoo Vu h-o=0- Therefore, yy = Asink,x GD) Yuko Yankee = Asink,e = De® ay! Wy ii) —4 => kAcosk, a Using the equations from conditions (ii) and (ii), we get =k, = k, cot (kl) = kf =~kl cot (kl) > B ‘acota _ _suvneeayectonteayoyrcti Correct answeris (a) 1 The minimum energy ofa collection of 6 non-interacting electrons of spin ~ placed ina one dimensional infinite square well potential of width L is [CSIR Dee 2012] @ i4s2n? /mi? (0) 91h? mL? (©) 7A md? (@) 3x7? / ml? ‘The energy of a particle of mass ‘m’ in a one-dimension square well potential of width L is given by ath? boo ea 2m 1 Only two electron with spin = can ill in the same state Soln. (corresponding to minimum energy) The minimum energy of the systemis, We ann 2ml? 2m? Lah ml? One! Correct option is (a) 1 ‘The energies inthe ground state and first excited state ofa particle of mass M = > in a potential V(x) are—4 and—1, an Gini sinsvbichsp. wy (x) =Wo(x)sinh %3 @ valshexa ‘The time independent jelfthecomssponding wnseetins are elated by cr [CSIR Dec 2012) 9 (X) =sech? x For ground state, Agyak tae Fortiexced sat pa aeons mene com Since, y,(x)= yo (x)sinhx s [-gs+v “fy o(s)sinh x=—y, (x)sinh x = SE (vels)sinnx)+ V(x) Wp (x)sinh x =—y, (x)sinh x ix = ~ va(s)coshae sis Me x ih. a o(x)sinb x }-vGa)vesinns= & ; = ~We(x)sinhx- ~cosh xe ~coshx te. sinh x- SEV Oy (x)sinh x =~, (x)sinh x One Dimensional Potential oO Se inhx tiat | one Dimensional Potential ayy oe “ = ~Peoshx He ~sinh x Fh + V(x) Wo (x)sinh x =0 dy, , 2coshx dy, GMa, 2e08hX Ao _ V(x )yy =a ams ax OO) Using the Schrodinger equation for ground state, we get yy , 2eoshx dy, _ d?ve 2eoshx Wed Wo 4 = was a) yy __y sinhx 0. _ 2sinhx 4, a een 0) Py cosh => Iny,=Insech?x+InC Therefore, the ground state eigenfunction willbe y,(x)=Csech?x ved Correct option is (¢) 9, "Let |0) and |1) denot ‘ound and the first excited states of 012] one-dimensional ((o)+[1)) {CSIR Dee 2012] @ Ax=Vh/2ma) @) Ax =Vh/4mo 1 Son, (x}=(ylgly) =f (ps5 gb) (0%) 10) at) (0e™) +(e) Cole) +2{(o1ata lo) + (1faalt) + (ofa) +(1fafo)) + ((010)+(A11)+ (011) (0) Therefore, Ax = (x?) ~(x)' = be Correct option is (a) Soln. One Dimensional Potential Consider the normalized state |y) of a particle in 2 one-dimensional harmonic oscillator: |w)=b,[0)+b,|1) where |0) and |1) denote the ground and first excited states respectively, and b, and bb, are real constants, The expectation value of the displacement x inthe state |y) will be a minimum when [CSIR June 2013] (2) b= by 7 =) ORB Ob Given : Normalized state }y) of a particle in a one-dimensional harmonic oscillator is |v) = by|0)+bo|1) ‘Normalization condition : b? +b3 =1 Here |0) — ground state |) — first excited state Expectation value of = b? <0|x|0> Since, x is an odd fu ion will be zero. Pa — : bby [< CAREER. ENDEAVOUR cee cece eras Therefor, sos= pr Cereb 6oM So, b,b, will be minimum if b, = 0 or b, = 0 Casel: — b, =O=>b,=1 Case 2: b) =0=>b =1 [Due to normalization condition b? +b3 =1] ‘So, correct option should be (a) In the answer key correct option is given to be (d) ie. b, = b, Ifb, =b, => b,b, will be maximum and its maximum value will be b? or b3 then, will be : : i z maximum and its maximum value willbe 267 |" or 203 [>= Correct answer Is (a) So Sol One Dimensional Potential it. Aparticle is in the ground state of an infinite square well potential given by, 0 for-a E,.At time ¢ = 0 the particle is prepared ina state (+ =0) 1 +q)- The Yr (Yi +4) shortest time Tat which ¥(¢ = 7) will be orthogonal to ¥(¢=0) is [CSIR Dec 2014) 2h @,-F) @) Soln. . Given: At t= 0, the The wave function of Since, y(¢=T) and a earl oer LNW. v cafgerendeayour com eth, vasa wo elBBIP yc gfe = Ar (Qnti)a T= Caciet For shortest time taking n =0, T= Correct Option is (b) On 4 Sol intial One Dimensional Potential nd E, )-The 2014 14 Soln. Let |v) =c,|0)+¢,|1) (where c, and c, are constants with c3 +c? =1) be a linear combination of the ‘wavefunctions of the ground and first excited states of the one-dimensional harmonic oscillator. For what value of c, is the expectation value (x) a maximum? [CSIR Dec 2014] 1 h L @ @ = (E Be © (1)= fae O73 h 1 h 1 © 0)" Vina "5 © Yona 3 a a E Given: |y’) = ¢9|0) +¢,|1) with gl? +leif = Expectation value of position = ‘The maximum valueof Correct Option is it ee www.careerendeavour.com One Dimensional Potential PRACTICE SET 2m Ifthe wave fimetion of a patticle trapped in space between x=0 and x = L is given by W(x) = Asi( 8 rE ), where Aisa constant, for which value(s) of xwill the probability of finding the particle be the maximum? [GATE 2002] L L L L L 3L ie . Eng E © and = OF OF OEMs @ za S The energy density of states of an electron in a one dimensional potential well of infinitely high wallsis (the ‘symbols have their usual meaning) [GATE 2003] @) Lim [ah (28)| (b) Lm/(nh VE) (@ Lm/[n/(2E)] (@ Lim /(2n AE) A free particle with energy E whose wave-function is a plane wave with wavelength ), enters a regionof constant potential V>0 where the wavelength of the particle is 24, . The ratio (V/E) is: 2 (GATE 2003] (12 ae ays Common data for [GATE 2004) ticle is maximum at repoesrearipee > s particle inte mlon Oto Fi is: Re P= ( Aimmotnonae ee SAR CER SNDE rn fosion y( 2)=u(x)exp(ikx), where u(x) is areal funcisi"This WitEYpORgS Tw SUCHE AREY” [GATE 2004] @ 2()v \ww.careerendeavour. cama, ) A one-dimensional harmonic oscillator carrying a charge-g is placed in a uniformelectrie field along positive axis. The corresponding Hamiltonian operator is [GATE 2005] wd? 2 wd? 1,4 +k +98 soot she? ~ gx Eth + abs © rae tye 4 wed? 2 ra ol Onin gt? dia? + gbx Fy Srt ge ae A particle is incident with a constant energy E ona one-dimensional potential barrier as shown inthe figure. [GATE 2005] ‘The wavefunctions in regions | and II are respectively fa\deravine oerillatare c re] 4 tial s(the jionof is: igure One Dimensional Potential qa 10. 1. 12, 13, 4, (©) oscillatory, oseillatroy c (©)oscillatory, decaying @ decaying, decaying, Aone-dimensionatharmonic oscillators inthe state v(x Falivel®) ~2y (x)+¥2(x)] Where yo (x), (x) and y,(x} ae first excited and second excited states, respectively. ‘The probability of finding the oscillator in the ground state is: {GATE 2006 3 9 @o OT On @1 A fiee particle s moving in-+x direction with a linear momentum p. ‘The wavefiunction ofthe particlenormalised inalength Lis: [GATE 2006] ~2L to - Where Y isa constant : : [GATE 2007] The normalization fi @ £ ‘The energy eigenval ve ots Mesto CBPERTEMMRESOGH COM fh @o wm on Om ‘There are only three bound states for a particle of mass m in a one-dimensional potential well of the form. shown in the figure, The depth V, ofthe potential satisfies [GATE 2007] Lay, anh? ee nh? anh anh? nih? we SOx”? @) Ar L, there is tumndla WY ORSOSHOATORTS UI LACAYA] and A, denote the amplitudes for the incident, reflected and the transmitted waves, respectively. Ay The ratio of the reflected to the incident ampiude ° A a) 1s (b) VI=T in magnitude | i | Aq) )_E | wath [asl |_ i (©) Areal negative number of [al ke | ‘Throughout 0 L. (©) For, x<0, the probability current is S,(I+R) (@) For xL, the probability current is complex. 21. Aparticle is placed ina one dimensional box of size L along the x-axis (00 as shownin the figure (GATE 2011] One Dimensional Potential ° vi 3 Vix) Ve 7E +— 3 For E 0 is ofthe form (@) (b) ee (©) ein Oem where cis areal positive quantity. 26, Aparticle is constrained to move ina truncated harmonic potential well (x> 0) as shown the igure. Which one of the following statements is CORRECT ? [GATE 2012} wl 3 tate iseven 1 nergy is ho 2 21. Gate, [n), given by [GATE 2014); | 28. Aparicle , is confined jefat ix =1, Another Particle P, is confined in a one-di al well with walls at x rmparing the two particles, one can conclude that | Sem egeaae ib — [TIFR 2010} t (@) the no. ofsodesin hep tent Rr eH R: | () the no. of nodes in the #Mekeit ea AB ie (©) the energy of the n'" level of P25 the SaneaS ThAPORPS== (the nee ofthe pela CaFERTEMEVOUT, COM 29. ‘The wave function ¥ of a quantum mechanical system described by a Hamiltonian f{ can be written as a | linear combination of, and , which are the eigenfunctions of 1 with eigenvalues E, and E, respectively | 46.43 , ; Att=0, the system is prepared isthe state Wy 5, += and then allowed to evolve with time. The | | th ‘wavefunction at time T= 74; willbe (accurate to within a phase). | E [TIFR 2010] 4 3 4 3 @s% see (6) ®, © gum @®, 3 4 3 4 ©s%+39, — 0-30, One Dimensional Potential & rtial 30. Two identical non-interacting particles, each of mass m and spin \/2, are placed in a one-dimensional box of length L. In quantum mechanics, the lowest possible value of the total energy of these two particles is ¢y If, instead, four such particles are introduced into a similar one-dimensional box of length 2L, then the lowest possible value of their total energy will be (TIER 2011] (@) 26, (b) Sey (©) 3ey2 @s, 31. A particle of mass m is placed in the ground state ofa one-dimensional harmonic oscillator potential of the form. V(x)= 4? 2 where the stifiness constant k can be varied externally. The ground state wavefunction has the form “hich w(x)ecexp(—ax? Vic) where a is a constant. If, suddenly, the parameter k is changed to 4k, the probability that the particle will remain inthe ground state of the new potential is (TIFR 2011] (0.47 (©) 0.06 (053 (067 (©) 0.33 0.94 | — =Ix! . 32. A particle in a oneélifnensional potential ae ) where a is a constant, It foot ITIER 2012] (@) Vox? 33. A harmonic oscilla i : P Vee . 1} where 9, (34) is the Me [+>]. The expectation i ' J) eP, is 8, oe value (£) of ene (TIER 2013] @ 1.58Ko — 146 ho 34, A particle is contin f gsi dtl 184 Vishingly thin but strongly repulsive : partition is introduced nthe.exact:centre of the box, and the-particleés’llowed to come to its ground state, then the probability jc i [TLFR 2014] come necoiiy enCateerenieaveurcom | ! masa stively ' ' | woot y ive? ‘ _ @ © ' | t Wweor i woot © t @ \ 1 One Dimensional Potential 36. 37, 38. 39. 40. ‘A particle is moving in one dimension under a potential M(x) such that, for large positive values of x, V(x) = ke, where k > Oand f > 1. Ifthe wavefunction in this region has the form y(x) ~ exp(—x*), which of the following is true? [TIER 2615] @a=Se1 () A=p (©) a=2p-2 wae ‘A one-dimensional box contains a particle whose ground state energy is €. Itis observed that a small distur- bance causes the particle to emit a photon of energy hv = 8 , after which itis stable. Just before emission, a possible state of the particle in terms of the energy eigenstates {y;,Y/z.-} would be [TIFR 2015] ya +, —Ayry + 5, V2y,-3y, + 5s ) a : (©) Val @ 6 1000 neutral spinless particles are confined in a one-dimensional box of length 100 nm. At a given instant of tie, i100 ofthese partici have energy 4, andthe remaining 900 have energy 225, then the number of particles in the left halfof the box willbe approx [TIFR 2015] @e2s jy,00 ‘A one-dimensional qu: [TIER 2015] requency «of the oscillator is denote the probability in each ccase that the system is uency change, which of the followingistrue? 2p, @ py The ground state (ap: ina one-dimensional potential V (x) is exp(-x? /2)}eoshy f2 aya), yrs 94 that ht =1,is (upto an additive constant). {JEST 2012] x x oF www.careerendeatourldain (© 7) Vietan (Bx) (@) 5 Vxcoth(V2x) ‘The grond state energy of 5 identical spin-1/2 particles which are subject to a one-dimensional simple har- ‘monic oscillator potential of frequency @ is [JEST 2012] (15/2) (13/2) , (1/2) @) () To ©) Ty (d) Sho tial One Dimensional Potential 8 0f] 41, A quantum mechanical particle in a harminic oscillato potential has the initial wavefunction w, (x) +, (x), A *) where y, and y, arethe real wavefunctions in the ground and first excited states of the harmonic oscillator Hamiltonian. For convenience we take 1 for the oscillator. What is the probability density of finding the particle at x at time t= x? (JEST 2013] ©@) (vi@)-vo0)? () (vi)? wold} stur. ona © (YO+ye0) © (viCoy' +¥«(@)) 42. The lowest quantum mechanical energy ofa particle confined in a one-dimensional box of size L is 2 eV. The energy of the quantum mechanical grourid state fora system of three non-interacting spin-1/2 particles is (JEST 2014] (a) 6eV (b) 10eV () ev (@) eV ae 43. Consider a square well of depth ~V, and width a with V,a fixed, Let ¥, co and q—» 0). This poten- tial well has [JEST 2014] (a) No bound states _.(b):I-bound state... (©) 2.bound states n.<_(d) Infinitely many bound states naa tor is each ofthe | \ ential ditive | shar- www.careerendeavour.com Angular Momentum and Spin 4.1. Orbital angular momentum : Tn classical mechanics, the angular momentum Z of particle is defined as z meee te {sino 2 )+ : wn cargsrendoavel ot 30, wa ‘Commmutation Relations [Ef [i..£,J=mé, [é,£,)=-ind, [é.4,]--iné, [é.£.]=-iné, i.e. the components of the orbital angular momentums cannot be measured simultaneously accurately. [2.2,]=0 [2.4,]=0 [2.i]-0 i.e. square of the orbital angular momentum commutes with any one of components of the orbital angul momentum, [ina]e0 [i-A.J-0 Angular Momentum & Spin [E., a Eigevalues and eigenfunctions of £? and £,: can be measured simultaneously accurately | Since, the commutator bracket [ 2°, £, ]= 0, therefore 2 and and both have simultaneous eigenfimnctions or eigenkets. Denoting the simultaneous ejgenkets by !2,m,) (where Vand m, ate the orbital quantum musaber and orbital magnetic quantum number respectively), the eigenvalue equation for ? and £, can be written as P\l,m,)=1(1+1)4 |l,m,) 1s, Wel jon is function of 8, 6 ie. where Yon (914) | where ¢ =(-1)" Raising and Lower | Raising Operator: wilwoabeerendeavour c corn ) Lowering Operator: Important relations: [énd, J=nd Actionof Z, and Z_: | E,jum) = (Em (m+ Dm, +1) je LY, (0.6) = (Cm) m, +1) AY, ,.1(8,8) yr Angular Momentum & Spin | Ar E I,m) = (04m) (T= #1) |L,m, 1) sol ie. LY,,, (0,6) = (4+ m,)(1-m, +1) AY, (8,0) Matrix Representation of the operators: Elements of 2 = (6 my sm) = e(e-+1) 12 See’ Sum, willbe non-zero for 1 =1' and m, an 0 0 100 B=|0 wo |=2n7]0 1 0| (orr=1) o 0 wm] loon Blements of £, = (€4mi'[Ee|,m) = mf See Sys, willbe non-zero for = 1'and m, = mie i Elements of £,=(¢' 4 willbe non-zero for 1=1 and m,=m,+1 Sol Elements of j,=(t' 2 willbe non-zero for 1=T and m, : careerendeavour.com i V2n\1 0 0| (for!=1) 010 Expectation values: (&)= (8)-(6)-[aeon-vi] ee ee Sol Example 1: Which of the following are eigenfunctions of L,? For the cases where the fimction is an eigenfiuntion Of L,, find the correspondence eigenvalue. (a) sind e* (& lor) (©) eB sing {d) r" cos Spin nition soln. Soln, We know, L, @ ning (sino ef) =(-in)sino (je =nsing So, eee isaneigenfunction of L, and the corresponding eigenvalue is a ng el) <(-any(i)et@r4) = nel) So, el! wo isan eigenfunction of L, and the corresponding eigenvalue is ft. © ~ns(e® sing)=~ine cosd So, e® sind isnot eigenfunction of L,, ay ME 94 So, r* cos@ is an Example 2. (a) Is values? sanaboms (a) We know that, i ‘Thus cos +sin é (osovsinoe wie a ans “] Thus, cos +sin 6 e' is an cigenfimnction of L? and the corresponding eigenvalue is 27. LZ, (cos +sing é) = cL¥0 (8.0) +c2L,¥] (0, d) = 01.04 cghY (8, 9 sind ef ‘Thus, cos@ +sind # is notan “eet Lely You can write Ypo(@, 9) as |2,0). Also, L,|£m)=Al(¢—m)(€+m-+1)|£m-+1) Soin. Soln. q@ Angular Momentum & Spin Ts, £,|2,0)=AV6 [2,1) a= etl 0) =) (8.6)= net (+ 1ca-S ho (6.8) = foe [St 3c0s? 6 ~ t+ ieot 0 (ost 8- i)|- ~ [sina cose Example 4, The operator associated with the measurement of the product £6 is (Lely +LyL,)/2, Calculate the mean value of £4 inthe state |é m) . Wehave L,=L,+ily and L.= Example 5. a mecspran Gerona 1) +}1,0)+|1,-1) J is (a) 2, 6 Be © 48, @o 4.) = 3[ (tales) +(1af2,ft) + (142,11) + (to, 12) + (tof, 1) + (oz, [1~1) +(1-1z,[tu) + (1=12,]10)+(1-2)2,)1-1)} L,, willraise the value of m to m + 1. Only those terms will survive in which this raised valued of min the ket is equalto the value of ‘nr in the bra. Thus, (ta) (eb9}olt—1)] $f (uA) «ofa = Ar ipin y2 eket Angular Momentum & Spin 4.2. Spin angular momentum : Stern Gerlach Experiment ‘The existance of spins first confirmed experimentally by stern and Gerlack in 1922. Using silver (Ag) atoms, Silver has 47 electrons, out of which 46 electrons form a spherically symmetric charge distribution and 47* electron occupies a ‘5s’ orbital (/=0). Ifthe silver atom is its ground state, then its total orbital angular ‘momentum willbe zero. thes Spin uy Beam — Spin down N Non-wniform magnetic field Ifabeam of silver atoms passes through an inhomogeneous (non-uniform) magnetic field (along z-direction), wewould expect the followis @ Classically, there yf sacra {ibe symmetric aout theundefeted direction 2 (@ Accordinh to S¢ ar momeritum then the beam will split into dise und state (/=0),therewill be only spot on the scree: 5 But, in experimentsit To solve this problem, to its orbital angular momentum , the electron possesse: ftum has no connection with the spinning motionof the electron fuected with an intrinsic degree of fal analog. Unlike orbital angular spaial degrees of eg about its own axis. 1 freedomie. spin. Consider particle AREER. CHDEARTTRS 1’, The magnetic dipole moment www.éaréefendeavour.com where [is the orbital angular momentum of the particle and z-components of magnetic moment will be For an electron the relation will be 42 =~ Ifthe electron is the eigenstates of £, ,then e =| Ina (4), (en pm where (2) = Wg = Bohr magneton and mis the orbital magnetic quantum number. Me Si Angular Momentum & Spin ah lt he. Fora particular value of J, we will get (2£+1) values of'm, ie m, veg Lyd Similarly, ifwe take electron as spinning charged sphere, then the magnetic dipole moment due to the spin motion ofthe particle will be where J isthe spin angular momentum oftheelectron and g, is the spin g-factor. Therefore, z-components of magnetic dipole moment willbe Ifthe electron is in an eigenstate of S, , then 2 fnere || — yp = wae ue te) (us), =-S.Ham, Fora particular value § Every fundamental Fermions: Parti! Bosons: Particles ha’ , protons, neutrons ete. r ete, : Photons, pions, ‘Therefore, there are “ @ Orbitalengutar and itis characterised by two quantum numbers iss ‘Thus, the total angula. ues from -s to +s, and itis characterised Ree ica z-axis ie. ‘m,’ which can take values from -/10+7. In the Stern-Gerlach experiment, the silver atom has orbital angular momentum to be zero ie /=0 but the spin angular momentum to be 1/2 ie. s=1/2. Therefore, total angular momentum will be 1a 2'2 =|!~s| tol! +s| For thiis, we are getting two spots on the screen, Pauli’s Theory of Spin : + Spin angular momentum has components Sys Sy and S, and the corresponding operators satisfy the foll | owing commutation relation: [8,,5, Jains, [8..5.]=in8, A Bpin vin tum rtwo take atthe re fol xs volar Momentum & Spin | (8, 8.) i.e. they have simultaneous eigenstate as following: §?|sm,)=s(s-+1)A?|s,m,) 8, |sm,) =m,h|sme) + Raising and lowering operators are defined as following: §, +18, 5, |s,m,) =hy(s—m,)(s+m, +1) s,m, +1) S.|s,m,) =Ay(e+m, )(s—m,1) s,m, —1) 8, 1 a 1 + For a spin > particles ike electrons, m, can take values 3+ and ~>, 0 the possible spin states are Angular Momentum & Spin + Pauli Spin Matrices: Spin-angular momentum can be related with Pauli spin vector as follows: ate 2 O11 0 -i (i 0) a where Ox =| ) y= o2= are components of the Pauli spin vectors and known 10 io {o.-1) 4s Pauli spin matrices. These matrices satisfies the following conditions: > \ Si Angular Momentum & Spin town Soln. Soin, () o2=02 =o? =1 (i) 0/04 +040; [oj,0,}=215 Gi [0.9% |= 226 gor (iv) Pauli spin matrices are hermitian, traccless and determinant is ~1 (¥) O00, =i (vi) Since spin does not depend on spatial degrees of freedom then the components of spin 5, Sy» Sz commute with all spatial operators ie, momentum, position, orbital angular momentum, (vi) PY = Feosa +io;sina A\(68 Example 6, The wave fintion ofan electron at an instants given wy =f (r,0)e" x19 .Caloulate the (vii) For any two vectors 4 and Bi ( =(48)1 +10 (4x8) ‘The average vend Now, cA The given wave fue is <2, This shows that it is an eigenfunction of L, shows that itis an eigenfunction of S, with eigenval 110) Lesmeeemmed Therefore, ve gareere|deavour com So, (vlaslv) =e [2 lv) +(v lv) ]=348 Example 7, The spin part of the wave function ofa spin-¥4 patticle is cos yyy» + sina: e!9%-12 The x« h component of the spin is measured, Find the probability of getting the result >. Here a and fare real constants, ‘The wave function before the measurement is | 71) =cosary,,. +8ine” 7p h, ‘The wave function after the measurement will be eigenfunction of S, corresponding to the eigenvalue = i.e. (42+ t12) Soin. Soln, <= Angular Momentum & Spin A So, probability of getting > inameasurement ofS, i | | ot 1 iB (alz)= Jp leosa rant sin ae x | 42+ X12) 1 ina et = pleaser aia |va)+sinae? (x yal z-u2)] I i = zz (cosatsina e ) ‘Thus the probability is Kalae 4 (cosc+sina e?l(cosa+sina e?) =1f142c08a sine cos 6 2 We can get the eigenvalues are aumple 9. Clete waliterdevoins com (LSv)|¥ (0,6) 402 ]= tal? (0,0) Stina 130 (0,0) =( AE) 9 (6,6) Hf La (G9) 1.1 (0.4)] =5[ ian 0.4) '(6,4)]--E [nk @e)+17'9)] and Sutin eA Theirs, (45,)[18 (0.8), Je ealM (.)+1@, )Jzrve Sol Example 10. A particle isin an eigenstate of J, Prove that (J,) =(J,)=0. Also find the value of (J?) and (J}). Soln. Let the eigenstate of J, be | jm) Spin | Angular Momentum & Spin (4.)=(imi(I45.)Lim)=2 mJ, m) +2 (jm J. jm) = I FD= mw hj jam ot) +2 (FFD) — mbm hj | j,m—1) = since (jm| j,m+1) = (jm| j,m-1)= Similarly, we can get (J,)=0 ‘We know that, J? +J7 Therefore, : i 5 It is expected that ' : sesare i Example 1. The Show that (A) Soln. (6.4)(0.B)=(6, A ith those of o. . =074,B,+054,B) +0,0,4,B, vod i Using the relations o dni Caredtendeavourcamn © 6,6, +0,0, =6,6,+0,6, =6,6, 4,0, =0 ‘We get (6.4)(.B)=(4.B)+i0,(4,B, ~ 4,B,)+i0,(A,B, - 4,B,)+i0,(4,B, ~ 4,B,) = (AB) +io.(AxB) Example 12, Find the equivalence of the following operators: @ S2S,S2; (di) S7S7S?; (iii) S,S,S? (J ose elo) 6 = Soln. (i) $25,5? =| io otte,(* 3 ‘ 2) Soln. The total spin angular momentum S of the two-spin system is given by Example 13, Consider a spin (1/2) particle of mass m with charge -e in an extemal magnetic field B. () What is the Hamiltonian of the system? (9 LES is the spin angular momentum vector, show that a € sy p) dt and [ 5,8, B,(~inS,i +ins, i) Ss 5.0.) (558, [5.52] Fg M(S,2. 8.8, )i+(B,S,~ BS.) 34 (5.8,-8,5,)F] Therefore, = de e £(548) Example 14. Consider a system of two spin-half particles, in a state with total spin quantum number S = 0. Find the eigenvalue of the spin Hamiltonian H = AS,.S,, where Ais a positive constant in this state Angular Momentum & Spin L Eigenvalue of 45,5; = a: (+) a4) oce/ore ~4 ‘Example 15, An operator P describing the interaction of two spin-hal€ particles is P= a+b¢,.c,, where a, b are constants, with ¢, and o, being the Pauli matrices of the two spins. The total spin angular momentum $= 5, +5, = (1/2)A(6, +6,). Show that P, S* and S, can be measured simultaneously. Soln. P, S? and S, can be measured simultaneously if (P, 7 ]=(P,5,]=[87.5,]=0 ‘We know that [S?, S,]=0. From the definition and [S,,P]=[5, Since $* and S, com ineously, ie Example 16. ovaitelelf fabfh magnetic moment 11 in an external magnetic field B, in gal is a fanother constant magnetic field B is applied in the -di ASM coal 3! "stn eo Heisenberg picture. a Soln, The magnetic moment of the electron is given by “8 “The Haritonian, iB = —1,B, = 150.8, For total applied magnetic field B = B,)-+ B,2, the total Hamiltonian H=u,(0.B, +0,B,) ‘From equation dui 1 Hat =o B, Ge eet Glos (6.8, + 0,8,)] Soln, Soln. gq Angular Momentum & Spin 18,3 +10,,0,]8,2+[6,.0,]B,2 =ZUillo B,-0,8,)8-0,8,5-+0,8,21 i AG oe e S x B]=-=3[Bxo] =<[Bx = hal [sx B]= at [Bxo] ae HI] Which isthe time rate of change ofthe magnetic moment, Example 17. The slate Cen i Algyels of the spin-orbit interaction Hamiltonian 47.5, The 2p state means s| Therefore, the corresth Using equation (ji)-(v), FaSener 3 GARE Hae Www.care! serendeavour, com 43 We ~ 2h -ye =-Ane D Example 18, The Hamiltonian of a system of 3 non-identical spin-half particles is H=AS,S,— BUS, +5,) 8, where A and B are constants are S,, S, and S, are the spin angular momentum operators. Find their energy levels and their dégeneracies. Writing S=5,+5,+5, and S,, For For +S, we have ieee S? = $2.45? +28,8, = SS; = 3 (8 Sa -S3) Similarly, S,.5, (Si, - 5 -S2) y AL Sol Spin tction nergy Angular Momentum & Spin Soln, since s,=1/2 and s, =1/2, the possible values of the quantum number s,, = 0 and 1. When s,, = 0, the possible values of $= 1/2 and 1/3. The Hamiltonian becomes H= 4S,S,-B(S,+S,)5, 4053-89-84 BS" ~83-8}) The energy is A itsabsn #1) 95,41) =9265, 4D) + 2M E(641)~ 54650 += 565,40) Since s, = 5,=5,=1/2, ” 7 +) + be [ac D~552+1)- 3 = | i fH where A and B are ms “ENE EY Tal of the two spins. Find the energy levels of the system. Total spin angular “at rere recom ae 5,8, -1(s?-s?-8? x 2) Since 5, = 1/2 and s, = 1/2, the possible values of s are 0 and 1. The Hamiltonian, 17 = A(S,, +8,,)+B(S,S,) ~ 48, +2(S?-S}-S2) The corresponding energy will be Bap 3.3 Ey, = Amp oH [ss+-2 3| 3 When s = 0, the possible values of m, = 0, then Bo =~ BR, By When s = 1, the passible values of m, = 1, 0,—1, then By, = 4h+ 7 Soin, Soin. Soln. ie Angular Momentum & Spin Example 20, Fora spin 1/2 particle, the expectation value of S,5,8, is int ine int ine oF o-> om O-% n _#/0 17/0 -4]1 07] _myi 0) 55. Eaae.=4I all ol ‘eh 1| (55,5.)=([535,5I¥) ler Lee @) 0.075 Eigenstate correspond vrampienn ance OAH RDER vg UR is in 8, sepresentation. What isthe probaitity MAMAS GALE ERROR AMOME ellen 0-[8P lof} 0 [9] aa-%, gat 2 0 2 1 sng longt 1 Probability offindingz-components ofits spin in~zdineetion = isi Example 23. Suppose a spin ¥4 particle isin the state |) = K A h What are probabilities of getting +> and ~ zit (iz component of spin is measured? " So Spin Angular Momentum & Spin (i) xcomponent of spin is measured? (ii) calculate the expectation valueaf S, ie. (S,) Soln, (Given: |g) oe alalll] 2a Probability ofgetting S, = tbat a the operator eft si i Jan. careerendeayoubieor) B [CSIR June 2011) wt Tons (@ Teos+ i6.B ©) Isinot+ coset @ Tsin2at +" eos 20t H, Soln, Given operator: _ Using the identity, e* = 1cos0+iG.Asind (where f is unit vector) , we get Ay ea a Bei eB . = 1008 wt +i6.B sin of = I cosa +i°—sinat Correct option is (b) Soln. Angular Momentum & Spin 1 al Ina system consisting oftwo spin = particles labeled 1 and2, let Sal) and §(2 7 2) denote the corresponding spin operators. Here 6 =(6,,0,,0,) and 6,,0y,¢, are the three Pauli matrices. [CSIR June 2011) ‘A. Inthe standard basis the matrices for the operators St” 81) and s{s(”) respectively, La 0) #(-1 0 wi 0) #(-i 0 @Flo -)4lo 1 OF 0 a) alo i o004 000 -i 01 (0 i 0 oF #10 0 i 0) lo O - Oo} wil o #/i 0 00 OF ‘ . @ 7 a 4/0 100 4/0 08 4/0 00 -i/ 4/0 0 018 i 0 0 0} ioo o ooigd oo10 B. These two operators satis g(2) (2 @ {990,99} € (No(2)_ githe(2) = o[sts?3090)] A. PauiliSpin matrices: BRE Correct answer is (c) B. [sss] =s1050(50 -—ss)"st50? fo 0 0 -i) (90 0 -i) (00 0 -i) fo 0 -i) #/0 0 i O)0 0 4 0) MIO O ~i Ojmlo io “4/0 -i 0 o/4fo i 0 0} 4/0 i 0 of4jo i 0 0 i ooo) ooo) tioo o) li oo Correct answeris (a) An Sol Sol Angular Momentum & Spin 129) 3, Ina basis in which the z-component , of the spin is diagonal, an electron is in a spin state _{a+i/V6) vl a ] The probabilities that a measurement of S, will yield the values f/2 and ~f/2 ace, respectively [CSIR June 2013] (@ 12 and 1/2 (b) 2/3 and 1/3 (1/4 and 3/4 (@) 1/3 and 2/3 Soin. The eigenfunctions of S, in a basis in which S, is diagonal are w)=(5 | for 2 eigenvalue and |4,)= aig () for 2 eigenvaue ‘The given state can be written as linear superposition of |g,) and|¢,) as (+i, 2 3 The probability of idl ' i 3 i q i and the probably a Correct answer Is 1 adel ABEE We Aspin —> partic CAREER bi f'S? and S,. Ifwe measure S, the Vi probabilities of getting ww careeren¢ oa our.com ICSIR Dec 2013] eee eo L3 @) 5 and > © Fm FT (6) 0 and 1 ORmMa Soln, 1 (hei Given : Asin 3 5 particle is in the state y = Vil 3 fi & Angular Momentum & Spin 9 iv 2 Probability of finding S, as 4 and ~~ are (Gj? =f and |c2)? ==. respectively. ‘Method-2: 1 (+i Initial state of particle = |i) “Fils 1 0 Final state of particle = “en and |, [I] Probability of finding S, as +4 and ~> are |(ve, wif =yyand (ve lvyf 3 respectively. Correct answer is (b) 5. Let iu, denote the eigenfunctions of Hamiltonian for a spherically syrnmetric potential 1). The expectation value of L, in the state [CSIR Dec 2013] Soln. v $[v. set ae ee. in usual notation), is [CSIR Dee 2013] @ wt -3) wo @ tins oma Soln. Given: vate. 1+ V5 v210~ Vitva,41) (te)=(V|bely) vail lvanal-vT aul ee +2.)] Sol Spin | Angular Momentum & Spin hs @D [3 5 vil [lv ) +p lwano) Svan) Now, we know, Ly nem = (E=m)(E4 +1) AY peat Mbp = [(e+m)(E—m +) nem > 1f3 8 & vit 4] 8st alv ay) 8 fn) 8 ao 1) tyne) i M5 5.(3 Vl) 13 5 vii 5 ‘tation <> Vn] =-~—|+2 2 Vvan-S 2 - ag als tages es ag 1013} 1 hand mh are respectively the fle in this state satisfies [CSIR Dec 2013] e+ Dnt 2 | Son. | "Rtapomnenmessiemr Meemean ‘nce, (i2) =(72 72 2 dy sine, (2)=(65)voWiteaGrevideaWvour.com 013) For a given /, m varies from ~1 to /. j2 1 2 (8), occurs when ms => (2), eee 2 2 2) hep (Ez) yy cccurs when mae => (2) San Correct answer Is (4) 8. Consider the normalized wavefunction §= ayy, asp +@,Y, Where Yj, isa simultaneous normalized eigenfunction of the angular momentum operators L? and L,, with eigenvalues 1(1 +1)A* and mth respectively. If 6 is an eigenfunction of the operator L, with eigenvalue A, then ICSIR Dec 20141 hi Angular Momentum & Spin Soln, Soln. 7 call = @4 v2 Oe © 4 a, a : i Given the normalized wavefunction: ¢ = 4, ¥y) +4, Yio +45 Vi-1 Now, £,g -|4 a2 Using the operations ofthe ladder operators ie. Levin = {l= m\(L m+ )RY int Lovin = J+ mM) m1) Vines = a Vita, Vora Vir] Correct answeris Oy e: Let 65(6,,0;,0,), vectors in three dimens Cee “ya careorendedV@em Seal © @b)I (@ layer Letassume, G=ai+a,jtak and babi +d,j+b6 ‘Therefore, [8.4,6.6]=[a0, tac, +qo,ho, +ho, +0, | # [a1 hoy |+ [41040695 ]+[arr, 02] +[ 420.40, ]+ Using the commutation relations, [0,0 ]=2io,,[0,,0,}= 210,,[0,,0,] = 2iay we get [4.6.5.6] =2i(ab,0, ~a\bj0, 4,9, + a,b, + ab, ~ 3020) = 24 {(a,b, ~ashz) 0, ~(aybs -ash,)0, + (aba — ab) 0;} Spin | Angular Momentum & Spin astant | | 6. satrix) PRACTICE SET Let L=(L,,L,,L,) denote the orbital angular momentum operators of @ particle and let L,=L, +iL, and L_ =L, ~iL,. The particle is in an eigenstate of Land L, eigenvalues ¢(é+1)4? and Ag respectively. The expectation value of L., L_ inthis state is: (GATE 2001] @en 208 @o @en ‘ 12. Anelectron is in a state spin wave function 6, = (4 } in the S, representation, What is the probability of finding the 2-componenet ofits spinalongthe -2 direction? =~ [GATE 2002] @075 (6) 0.50 (00.38 (0.25 Ina Stem-Gerlach experiment, the magnetic field is in +2 direction. A particle comes out ofthis experiment in |+27) state. which: ofthe following statements rue? [GATE 2002] (2) The particle has : (b) The particle has entum. (c) The particle as turd. (@ The particle has $e x56 fF momentum. [GATE 2003] (@) 0,0,07/4,n7/4 FE 9-/4,97/4,0,0 The commutator, [i Yin (O.¢)is 2 spher 2 i (@) e(241)h f ~ ad) +£h Foraspin 1/2 partie, 8, are spin operators, is eC , [GATE 2003] in Sep romnnenase moma earn i oa wiweareerertldavour.con? “1 The commutator [L,,y], where L, is the x-component of the angular momentum operator and y is the y- component of the position operator, is equal to: [GATE 2006] @o (®) inx (6) ity @ ine ‘Which one of the following relations is true for Pauli matrices? {GATE 2006] (a) 5,9, = 9,0, (b) 9,9, (0) 9,9, Ie @) 5,9, =ic, Let |¥¥q) denote the ground state ofthe hydrogen atom, Choose the correct statement from those given below: [GATE 2008] @ [Luly ]Ivo)=9 © Fy)=0 © [LS]lyo*o @[S,.8,]}y.)=0 Angular Momentum & Spin 10. ul 12, 13. 45. 16, 17, 18. 6; (i= 1,2,3) represents the Pauli spin matrices. Which one of the following is NOT true? (8) 5,5, +60, = 28, (©) Theeigenvalues of 6, are +1 For a spins particle, in the eigen basis of $7,5, the expectation value (sm Ist |sm) is [GATE 2010] (b) [s(s+1)-2m* |x? () [s(s-+1)~ ~ IfL,yL, and L, are the x, yandz components of angular momentum operator L, thecommutator [ L,Ly, L, Which one of the following commutation relations is not comect?carmicwr A spin-half particle is ip and spin-down states. If tty and i) are the cigen to one decimal place, of the operator 100, +50, If Z is the orbital an, then Z.§ does not commute Wis! El ndcovoure com 0 } and () represents the spin-up and spin-down states, respeetively. When the system isin the state y, its probability to be in the spin-up state is If L, and L_ are the angular momentum ladder operators, then the expectation value of (L,L_ + L.L,) in the state |/ = 1,m= Let Zand f be the angular and linear momentum operators respectively, for a particle. The commutator A 19 20 ai 23 4 Spin | Angular Momentum & Spin 19. I 20. 010) rh} | on, I 1 2. Iti) of the q mute U] i a | 23, sin the | LL) q autator a ‘The Pauli spin matrices for three spin-1/2 particles are 6, and G, respectively. The dimension of the Hilbert space required to define a operator O = 6,.(6 x3) is [GATE 2015] ‘An operator for a spin-1/2 particle is given by 4 = A6.B, where B= ali 5), & denotes Pauli spin matrices and 4 is a constant. The eigenvalue of 4 are IGATE 2015] AB @*y () 2aB (©) 0,4B @) 0,-aB Consider the Hamiltonian H= fez Here 3 is the position vector, fis a constant and G =(0,,0,,0,), where o,, 0,50, are the three Pauli matrices, The energy gigenvalues.are,, aeRO [TIFR 2014] ee “1 @ f(\verrs iy) © #6 @ af (x+y+2) Ina Stemn-Gerlach| nt waithSpi : i |to form two spots on the screen, one directly above er. TI ef iio fe fereen at the position of the upper spot. The particles {hatgo ti ole are then passed t! ther Stemn-Gerlach apparatus but with its magnets ro 90 unt of the beam direction, Whichof the following sho appei e. (TIFR 2015] 4 -e+— e-| @ | gy www.careerendeavour.com An rigid rotator has the wave function [TIFR 2015] V0.9) = N[21¥, 0(,9) + (2+ 0%, (6,9) +32¥,(8,0)] where ¥, (0, ) ate the sphereical harmonics, and Vis a normalization constant. If Z, is the orbital angular momentum operator, and L, = L, 4 iL, the expectation value of L,L_ is 210? 23h" 2sn* oA om OA @o Define o,=(f'+f)and c,=-i(f*~f), where the o' are Pauli spin matrices and f,f* obey anticommutation relations {f,,f}=0,{f,f1 =1}. Then o, isgivenby [JEST 2012] (@) fi fnt 2st f-1 © aft fat Oss 25. 26. 21, 28, 29. 30. Angular Momentum & Spin Consider a system of two spin-U/2 particles with total spin 3,, where s, and s, are in terms of Pauli matrices o, . The spin triplet projection operator is [JEST 2012} dass ») 3-5. S45. _ @) gts 0) 47-55% © F7t5e O75 a ofomogeneous magnetic fed ofrmagnitudeB along z-axis which is prepared initiallyin a state [Y) = ll") |+)) at tinet=0. At what time‘? wil the particles bein the state |v) (ay is Bohr magneton)? (JEST 2012] to Zh 4) 022 fae a @'" TB TB © 2u,B (@) Never If J,J,.J, are angular momentum operators, the eigenvalues of the operator (J, +J,)/ hare % ae, [JEST 2013] (2) real and discrete wit ional spacing the L basis of states with m= ‘after the measurement? {JEST 2013} 1 ‘eal (@))0 a 0, v2 J What are the sinonihnvenioas@OFend@Se oun OOM inatin and gis vector? [JEST 2013] @a,+a,andc, (b)a,+a,andia, (0) #(a,+0,+0,) (@) £ Consider an eigenstate of Z* and L, operator denoted by |é,7m).Let 4 = ff denote an operator, where j isa unit vector parametrized in terms oftwo angles as (n,.",,1,) = (sin8.cosg,sind sin 4,cos@) . The width 44 in |é,m) state is: [JEST 2014] @ LC peos0 () {Gre cos (©) yele+i}-mtsing (d) fe(2+1)=m* hos Spin "Pauli which vetate, Angular Momentum & Spin = 1 31. Suppose. spin particles in the state y=]! vi Ve 2 If S, (x component ofthe spin angular momentum operator) is measured what is the probability of getting to [JEST 2014] 1 2 = 1 On oF © oz CAREER ENDERVOUR, putsansacemonmmenmn es www.careerendeavour.com Three-dimensional potentials 5.1 Three- mensional rectangular box : We consider a free particle of mass m confined to move non-relativstically in a three-dimensional potential box ofsides a, band parallelto thex, yand z-axes respecte Since the prices fie, the potential insiette always a positive pani js ecrnmemmzensrersce Fi tv be the velocity, tht CARE ERG Since the potential mau ais eere daycare time-independent 3- | : D wave equation to obtain the wave functiuon y (x,y,z) anidthe eigenvalue ofthe energy oftthe particle, | ‘The three-dimensional potential box has the following characteristics: (ayzya{ BrOa, y>b, z>0 a The wave equation for the Particle: ‘The wave equation deseribing the motion ofthe particle canbe written as 2 Ay =- ivy =ky Q) albox de the aich is 2 Three Dimensional Fotentiale & As V(x,y,z)=00 at the boundries and outside the box y(x, y,2)=0 at the boundaries and outside potential box. The boundary conditions can be writtenas, ee eaeamcecae y=Oandy=b zeDandzee| Using the separation of variable technique, we caan assume the solution of equation (2) to be ¥(sy.2z)=X(s)¥(y}Z(2) 6 ‘Substituting equation’ 2 in equation (2), we get yet Say th, xY ae dy? Dividing throughout by XYZ, we get @ such that E=E, ae Therefore, equation o (6a) (6b) (6c) Solutions of equations (6a), (6) and (6c) are X(x)= 4 sink,x+B,cosk,x where k, ¥(y}= 4, sink,» +B, cosk,y whete k, = Z(z)= Asink,z +B, cosk,z where k, = Boundary Conditions: ey = Three Dimensionat veraioa) X(x=a)=0 = Asink,a=0 =sinka=0 3k, = 2% (1, =1,2,3. a =0>B,=0 =0 => Asink,a=0 sink, (ii) Z(2=0)=0 > B,=0 nt (1, =1,2,3. Z(z=c)=0 = Asink,a=0 =>ysinka=0 =k, = a Therefore, the solution will become ne ¥(x,342) = 4,4 Ay sin xsin™ ysin 2 @ ne 2 ¥ (5912) = Asin xin “Z ysin a ‘Now, the eigenfunctio v(x, ‘Therefore, the total i ene ‘The eigenvalue of energy of the particle is, Je) The integers 1,,1,, 2, are called quantum numbers which are required to describe the stationary state ofthe particle, Since for any of 7,,n,, 7, being zero, w(x,,z)=0, ie., the particle is absent in the box. But physically this ismot acceptable and hence any of 7,7), 7, cannot be zero. For the ground state, , =, =, = 1 and the ground state wave function of the particle is, \— PB gin ®® gin 29 cin 2 Yauilr#)= oR sin sin sinS | @ ®) ‘Three Dimensional Potentials ofthe But and the energy associated with the ground state called the zero point energy is given by, 5.2 Three-dimensional cubical box: Ifthe potential box be cubical instead of rectagular, thena = = c the eigenfunction corresponding to the state of the particle will be represented by and the conespondi Ground state (in, ‘Wave function: ¥; Momentum : 71, www. careerendeavour. co ne my a, Wave function: Yau = Sin sin sin 2 17 7s 12 vay = fein sin in 112 Bmx. ny, nz Vina = fy sin sin sin = BOs eo mh Momentum: Pass = P21 = Pia =" V6 2q2 amine, 12 at Energy: Foi = Fi24 142 Soln, Three Dimensional Potentials Degenaracy Corresponding states (n,n,n,) 2 pré S11), (151), (115), (333) 26 =6 (433), (413), (341), (314), (143), (134) p=3 (422), (242), (224) =3 (332), (323), (233) =6 (421), 241), (124), 142), 214, (412) =3 (331), (133), (313) =3. GID, (4h, (14) =3 (322), 232), 223) Energy 7 gt | 1 P=6 (321), (132), (223), (312), (231), (123) in the units of = ( 2ma’ 1 p=i (222) " —— p=3 Gil), a3», 13) 9 p=3°— @21),(122),@12) Degeneracy: i The property that, twé different eigenfunctio Example 1.. aoe inthe ground ate and Ground state: n, = 7, ceca cn ail ererendenourcon th pth ph 24 Ry at = B= Bs. x10 ig - tay SENG = NB = SNF 5.1210 hg m/s Energy ofthe electron is ap? Egy FE a 181x10 7 =113 27 me First excited state: ee 2140 121 ie ah 24 Momentum ofthe electron: P21 = Piz ~ Pui =~ V6 = 8.1310 kg —m/s Sol Sol Sol 4) 12) 2) sand rethe ‘ctron eee ea) Soln. Soln, Soln. wen © 2ma? Example 2. An electron is confined in a cubical box of side 1A. (i) Find its lowest energy (ii) what is the ‘temperstture at which the average ofthe molecule of a perfect gas would be equal tothe lowest energy of the electron (k= 138x107 J/K)? (i) Ground state energy of the electron 6=362x10° J = 226 eV Energy oftheelectron: Fp... = ia, 3a? (1.05410) 249.1 e107 x(10)° (ii) I€T K be the required temeprature of the molecules, then =18.03x107 8 cle isin an eigenstate with energy @ ee Example 4. Find the number afenergy states and energy levels inthe range E <[15h? /(8ma*)] ofa cubical box of side a For a particle in a cubic box of side a, the energy is given by Foo (ni +n 6m) == (m2 + +m) 2ma' Comparing with the given expression, we get 2 +n? +n? <15 The number of possible combinations of (7,7, ,) is (it 1 way 12,02),@11) 3 ways (113),030),61) 3 ways (122), 212), 221) 3 ways 22) 1 way eS ‘oe Deronaional Potala Soin. (123), (132), (2.13), B21), G12) 6 ways Total 17 ways Hence the No. of possible states = 17, The No. of energy levels = 6. Example 5. A crystal has some negative ion vacancies, each containing one electron. Treat these electrons ‘as moving freely inside a volume whose dimensions are of the order of lattice constant. Assuming the value of hitice constant, estimate the longest wavelength of electromagnetic radiation absorbed by these electrons, ‘The energy levels of an electron in a cubical box of side a is ie =F anton), ma 2ma’ ” Lattice constant a= 1 A = JO"! m, The energy of the ground state is given by =1,2,3,.4 795x107 J. ‘in = ma? 2 steerer ntgoSE The longest wavelengi yyy, and hence 2x6 Fy, = BEX 2ma* We considera particle (CAREER ENDEAVO IR soap SES Ds SEU R along thes), componente diplreanet desea: sy rovk, sk, are the force constants and ©,,@,,0, are the angular frequencies along x, y, axes respectively. Such a systems called a three- dimensional harmonic oscillator. Again, and F,=- 2 where the potential energy Vof the oscillator i, 1 2 V(x,9,2)= gl’ +h, y +k,2") ‘The time-independent three-dimensional Schrodinger equation for the system is # Fv (sy2) +5 (bes? +h? hz") v (5-2) = BV (02) \3 als tants wee- ‘Three Dimensional Potentials % [2 (aix? +a3y? +032") ]y =0 0°) me VF where, ape Equation (9) can be sotved by the method een ‘of variables. The energy eigenvalues and eigenfunctions ofthe three-dimensional oscillator are a E -(n-4 +5 5)po. E, -(1,+4}p0, where, Mgy m, and X(x)=N,, Therefore, the co aE: Forthe _ REEL AE com sse[nctyene3fion(oe3)pe where =n, +n, +n, is the total quantum number andn=0, 1,2,3, It can be shown that except for the lowest energy level (n, =n, =, = 0,ie.n=0) allthe energy levels are degenerate, ‘The zero point (or, the ground level) energy B, for 11, = 3 Sho 2 ‘Thus the zero point energy ofa three-dimensional isotropic harmonic oscillator is three ies thatfor the linear harmonic oscillator. the ground level f the oscillator is thus non-degenerate, = 2, = 0 ofthe oscillators given by S a The state with principal quantum number n= 1, there are three levels each of energy 5 %@ 2 Three Dimensional Potentials So, the first excited state is 3-fold degenerate. Similary, the state with n=2 is 6-fold degenerate. Thus all the energy levels of the isotropic three-dimensional oscillator are degenerate and the degree of degeneracy ig Flme(nea), Figure shows some of the energy levels ofthe oscillator. Sho 2 Sho 2 n, n, n,|n| Wave function] £, | Degeneracy(p) 0 0 O}0]~ veo Sho P Te0Rs0 Vioo o 1 ofl Vouo ry ae RY eS CS — t 1 1 She 6 i 3 % q o im Yoo. DEAVOUR RGMEEN{3"0;0); 073; 0), (0, 0,3) GP 0.2) 2,0,1); 2, 0), 0, 1,2) (14) n=2 aaa (2, 0, 0), (0, 2, 0), (0, 0,2) 1,1, 0), 4, 0, 1), 0, 1,1) | __B=1__ (1, 0, 0), (0, 1, 8), (0, 0, 1) States 3 00,0) States 1 States 10 States 6 ia tials first level wey is ‘Three Dimensional Fotentiale iS 5.4 Spherically symmetric potentials : We consider a particle of massm moving in three dimensions under the infiuence of. force exerted by another particle located at the origin. The force is represented by F =-VV (7), where, 7 is the distance of the particle fromthe origin. If V(r) depends only on rand not on the direction ofr. Such a potential, which depend only rand not on @ and ¢, iscalleda spherically symmetric potential. ‘The time-independent Schrodinger equation, fora particle moving ina spherical symmetric potential, n polar coordinstesiis given by V?(r.0.8)+ Zev (nw (r.06 Now, y? oe = ay vat 9 where, R(r) and Using equation (13 We get, ‘www. wa career payor com O06) sin2@ Og? Se le-V(n)]=0 (14) Tnequation (14), the second and third terms are: oS ‘ofr whereas the first and fourth terms dependent onr, So, wevan write, 1 [z 2 (sno 24)) Q(64)| sind 20 20 as) 1 df par 2naGee . and Roa" 2) DM a6) where, Cis a constant. ‘Three Dimenelonal Potentials ‘Solution of the Angular part: Equation (15) is independent of V (r), so the solution giving the angular part ofthe wave function y (r,8, 4) will be the same for all spherical potentials. Further, equation (15) is related to angular twomentumin the following way: 2 (ino 2-\4 1 Fine Sarg r af... ano, Fae wean let - =-#C2(6,4) Therefore, 72(6,6 )= 42(8,¢) ” Clearly, equation (17) is the eigenvalue equation for the operator Z? -(2(@,¢) and 2, arethe eigenfunction. and eigenvalue respectively belonging to the operator 72. Thus the normalised f Ye (haga 3 here 0< mse. Some of spherical har For£=0: (64 For For é#2: “a 4) sin cosbe"# (ar 12 scot $Y!” 3c0s 6-1 #2 (0, (2) 5 1s? #e¢e(2) sinOcos6 e#* sua 1s ~ AS) sin? 9 e-™# co ~(o.0)=4/ dale | three Dimensional Potentiale Nonmalization condition of the finetion ¥;" (8, 6): 9.9) 1 nthe J J Mn, (6.4)¥in, (8.6)sin6 40 dp =1 a0 Solution of R(r) equation: The radial part of the Schrodinger equation is, 4 pa 2m ? E-V(r (41 “gael GEV Olec=—eea 1d 2dr), {2m £(E+1)] =x9(r 2). {ee [E-V(r)]- = fa=0 (18) 1 Now, v(1.8,9) = R(r)©(8)©(4) = R(r) (0,4) =R(r) YP" (8, 6) ‘This is the complete three-dimensional wave fiction ofthe Schrodinger wave equation for a particle moving in ction a spherically symmetric M(). For a bound system, the wave function must satisfy the condition that (r}>0,a5 r—>20 “ pcmemonromnants IV (r) isknown, thé, sxplicits a jince equation (18) depend onn and Jean have a discrete Feros e2 r alu jscrete say, Ey, Ef, complete wave fundtigh E written as Here will be (2£+ 1) wave function , |) fold degenerate. The quantities umber and azimuthal quanturn 5.5 Hydrogen ferent states, “The simplest of the An Se sheelyG ih BUR eto moving around the nucleus containing only one it nee inder.a central Ais Sut of the Coulombian electrostatic attractive force. It Saaran -omplex atomic systems containing more than one elect PALE TET IRB HISCeNE Bohr (in 1912) on the basis of hydrogen atom, Schrodinger gave the solution of the wave equation for hydrogen atom is 1926 formulating the wave mechanics. There was extensive development of quantum theory of the hydrogen atom by the subse- quent workers, viz, W. Heisenberg, max Born and Jordon and Pauli. The results of hydrogen atom was successfully extended to other hydrogenic atoms, The hydrogenic atoms are oftwo types: 0 Ionised atoms containing only one electron moving around the nucleus, which may consist of more than one proton and neutron. Singly ionised Helium (He'), doubly ionised Lithium (Li), triply ionised Beryllium (Be) are some of the hydrogenic atoms, i) Atoms of, ions with closed shells and only one electron in the outermost shell form the second type of hhydrogenic atoms, Atoms of alkali metal and alkaline earth ions are the examples, ‘The wave equation for the hydrogen atom: ‘The hydsogen or, hydrogenic atom is a two-particle system consisting of an electron of charge—-e moving round the nucleus ofcharge +Ze, The electron inthis system moves under the central fore arising out ofthe Coulom- ic electsnstatic attrantion hahween tha nniclane and tha alanten tow al (st Three Dimensional Potentials ‘The potential energy of the system in absence ofany external field is, The potential energy is indepenent of @ and ¢ and thus spherically symmetric, Reducing the two body prob- Jem into one body problem, the 3-D Schrodinger equation in spherical polar coordinates can be written as 1 6/ 2)éy a 4 Sh) ow zal 6. + E-l V(r 0 Fan Var end 008° 56.) ant ap? * LEV = Marcelle +m, where # = is the reduced mass ofthe system . Using a technique, one can find the soluion ofthe above equation to be (18.9) =Ru (On (8), (8) W/careérendeavour, com Pr pita ya 30 aay 27- tert |e re B\ a) ay a5 ) ala) lee ia ‘tials ‘Three Dimensional Potentials = 2 oe Rulr) nas Qo Rulr) 0.5 0 o4 ' Ry (r) 03: ot \¥ é t 10.1 = ° fs I ‘Table: Real formo! som (Exo 5 tron) for various values of, land mn - | n tm (Si TOG ase aro | www.careerendéavour.com os i 200 2s Ye = ae gee)” i PP sind o 21 1 2p Von wear? ? sind eff | 210 2» V0 =—2— te pe?” cos 9 abn a? 21-1 2p vos leap moe | 2 | Va00 = ane 1 (21- 18p +2p?)xeP? i 300 3s 2 when, p= 22, a, = & G&G Three Dimensional Potentials ‘Radial probability density : ‘The probability of finding the electron in volume element dr is given by Waal dt =|Vyiql° 17 sinOdrdede ‘Therefore, the probability of finding the electron ina spherical shell ofradius and thickness dris, au P(r)dr= J | Wael 7? sin@drdOdd =(R, fdr seogea Here, P(r) is called the radial probat ty density. 28 Figure : Variation of Rye (1) with rin units of@, Energy values forthe hydrogen atom: Energy eigenvalues of n state of hydrogen atom __ Hae 32 ehn® Sob ale | Taree Dimensional Potentials ss =~ 32 ern where, n= 1, 2, 3, ......... and is called the principal quantum number. ‘This shaws that the energy values for different quantum states discrete which is in agreement with old quantum, theory and experiment. Degeneracy: For a particular value of m,€=(n—1),(n-2),~-2,-1,0, ie. (n) values and for a particular value of J m=0,(£-1),(€-2),.. —(€-1),-£, ie., (2€+1) values. Thus degeneracy of particular en- exgylevel is, >,(2/+1) =n? Coulomb + Central field ‘ Non-central field m field! ; i i 12 : + t 0 1 a t —-2 4H 7 0 1 ae 9 states t forn=3 1 0 1 ' 4 ' i 1 : a \ ie 4 states ‘ REE forn=2 \& _ seo a 1 Wwww.¢areerendeavour. com : ' ' | ; ete I=0 ' n= ae forn=1 | Figure : Degeneracy of hydrogenic atom Example 6, Show that the most probable distance by the electron from the nucleus in the ground state of | hydrogen atomis equal to the Bobr’s radius. Soln, For the ground state of hydrogen atom, n=1, €= 0 and m= 0. Hence the ground state wave function is, \n Ly Yia0-| “a | ae (may Since, the wave function is independent of @ and ¢, therefore, q@ Three Dimensional Potentials Radial probability density= P(r’ J=|ioul? 4ar? For most probable distance, P(r) isto be the maximum E(ee2a| ae al P| 2 |e 4 are 0 | 9 5 ( =0=r=4, al \ a0) ° ‘Thus the most probable distance of the electron from the nucleus of hydrogen atom in the ground state is equal tothe Bohr radius (ap ). Example 7. The normalised ground state wave function of a one-electron atom is given by rr an( 2)" v(,8,9)=2° “(2) €7' here the notations have usual meaning, Evaluate the probability of the Soln. | LOAREER ENDEAVOUR) chuamecrmitacner Tse ‘Therefore, the, proba eg the electron inte normalised en state of one-electronatomat a argerendeavour. A Zz Example 8. Ifthe ground state wave function for the hydrogen atom is distnce atomat adistance greater th yet en ve a show that average distance of the electron from the mucleus is 1.5a,. Soln. We have the average distance of the electron from the hydrogen nucleus in the ground state is, ria, LL grange 5 fre? 21/00 2 sin Odrd dd 7a | \3 Sol Sol | uals | three Dimensional Potentials 3) Using the standard integral Jetx'ae =T, we get fp ‘a fanoao ap= x 2820 na 9 a a, (2/5) 2qual 3 ‘Therefore, (") = Faq =1 Say Example9. Find the expectation value of potential energy of the electron in the hydrogen atom in the 1s Ate state. The Wave function for the electron is Is stateis given by ofthe aata ® Where, do ==" isthe Bohr radius Soln. The potential onal Therefore, the exp. Soln, But, 1 lanes a ore sing 30-00 r*sin? 6 ag a. 8 Since Viq9 depends on r only the terms in = Sand 3p ate ze. jend( S23) : Soln, Soin, Soin. Therefore, (Ex) = Example 11. Estimate the uncertainity in the radial position of the electron in the ground state of hydrogen atom, For ground state, Uncertainity inthe ra nanos soeeo e Example 12. Find o i ofa hydrogen-like at ‘The radial probabil Ry = constant x exp| , wil be maximum ‘yoww, cateerendeavour. com $x gcinda| 27-22" Jo dr \ % a ar Similarly, dP, / dr =0 gives r=9a, /Z In general, r, Fnax =I, |Z Example 13. A positron and an electron form a shortived atom called positronium before the two annihilate’ to produce gamma rays. Calculate, in electron volts, the ground state energy of positronium ‘The positron has a charge +e snd mass equal to the electron mass. tials ‘rogen ‘bitals ihilate ‘Three Dimensional Potentials Soln. Reduced mass for the positronium atom, is met, My 2m, 2 where m, is the electron mass. Since, the energy for the tydrogen like atoms is directly proportional to the reduced mass. Therefore, energy of the positronium atom is half the energy of hyécogen atom. 6 Therefore, the ground state energy for the positronium atom is ~ Bey =~6.8eV Example 14, At time ¢= 0, the wave function for the hydrogen atom is 1 7,9) = = 2Vi00 + Vato + Y2Vn + V3V 2,4 ver,0) Tio 10 + V: Ms: War) ‘where the subscripts are values of the quantum numbers m, /, m. (i) What is the expectation value for the energy of the system? (ii) What is the probability of finding the system with /= 1, m= 1? (i) The expectation — 1 =A (48, +8, +2 yb Es +2 Since E, = -13.8 @) CAREER EMRE: (ii) The required prSbibiiyispvenbysBn (21 www. careerendeavour. com Less 4e Soln, Soln, Soln. a Three Dimensional Potentials CSIR PREVIOUS YEAR QUESTIONS, The energy levels of the non-relativistic electron ina hydrogen atom (i.e. ina Coulomb potential V(r) o —1/ rare given by E,, 0¢—1/n”, where nis the principal quantum number, and the corresponding wave functions are given by Vain.» Where ¢ is the orbital angular momentum quantum number and m isthe magnetic quantum number. The spin of the electron is not considered. Which of the following isa correct statement? [CSIR June 2011] (@) There are exactly (2+ 1) different wave functions Yin» for each Ent» (b) There are £(€+1) different wave functions yn, f0F each Ey (©) Exim does not depend on ¢ and m forthe Coulomb potential. (@ There isa unique wave function Wyrm, ANd Egy . jueneracy 1(neglecting spin) that arises due to the sphe nly on radial distance ‘of the electron fromthe nucle Correct answer is (¢)s fan electron is in the gt its distance from the protonis, more than one Boht radi (CSIR June 2011] Probability for electro: e 2 P= flvioo (8) Ane fy Correct answer is (a) Given that the ground si ffigho OAR GGTOM GF OBYS NYO ENqund state energy of positronium (which isa bound state of an electron and a positron) is (CSIR Dee 2011] (2) 6.8eV (b)-6.8 eV ()-13.6eV ()-27.2ev For positronium atom, reduced mass =) Snr = 9 Since the energy eigenvalues of hydrogen lie atoms is proportional to reduced mass 1 ofthe system, ‘Therefore, energy ofthe positronium atom E, Ground state energy of the positronium atom Ey = — Correct answer is (hi. sia tials ) three Dimensional Potentials aq 4 ‘The energy of the first excited quantum state of a particle in the two-dimensional potential /rare V(x, y)=pmo? (x? +4y?) is: ]CSIR Dec 2011] msare 3 5 @ 2hw ©) 3h0 (© 5h (@ ho sntum wom | 5! Given potetia: V(x4y)=4on0?x? + 4m(20}" y E, Dy , 2 : e(n+4 L sama cpemant (1.+2}too(o,+!} 000 Ground state : First excited state: n, =1,n, Correct answer's (d) athat | 5, A particle of mass ms (0Sxhd Given: 8,4, = 5h@ ‘The corresponding Yaa, (9) = 40% Voil%y)= Ah = Valry)= Correct answer's An electron is in the g approximately equal CHE E (a) 0.60 'b) 0.90, 1) 0, 2 nal rhareereneasyour cont?” Loin whee, Wo" Tere “ on 5 aot femesdrel-S may é Er TITBEANOIE sitar se Renae seit = |vivetnrdr, 3 32 Correct answer is (d) ‘Three Dimensional Potentials The spinless particle moves in a central potential V (r)) . Which of the following statement is TRUE? [GATE 2001] (a) The kinetic energy and the potential energy of the particle cannot have simultenous sharp values. (b) The total energy and the potential energy of the particle can have simultanous sharp values, (©) The total energy and the square ofthe orbital angular momentum about the origin cannot have simultanous sharp values. (@) The total energy of the particle can have only discrete eigenvalues A quantum particle of mass mis confined to a square region in xy-plane whose vertices are given by(0,0),(L, 0), (L, L) and (0, L), Which of he following represents an admissible wave function of the particle for! m, n poste integers)? [GATE 2001] @ asin oe oo Zs oe 22) rt Common data for’ [GATE 2003] The sum of he energi (a) 10n°A? mL? The degeneracy of te! @! 2 Ina hydrogen atom, the Becker Ty fork [GATE 2004] @4 We @a, The degeneracy ofthe: AREER EMBEA Sat [GATE 2005] 4 Mu. careereggeavo r COm 0 A particle is moving in a spherically symmetric potential V (r\= arr? where a: isa postive constant, Ina stationary state, the expectation value of the kinetic energy ofthe particles [GATE 2006] @ (7)=(V) O(r)=2r) —@(T)=3%7)_— @ (7) =4(r) Anatomic state of hydrogen is represented by the following wave function: war) fam Where a, is a constant. The quantum number of the state are [GATE 2007] @ ¢=0m=0n=1 (b)£=lm=1n=2 (©) ¢=Lm=0n=2 (4) f=2,m=0,n=3 s\3 12 als, ‘nous (Ly mn ‘Three Dimensional Potentials 10. 12. B. 14, The radial wave function of the electrons in the state of n= 1 and I= 0 in a hydrogen atom is, R, 2 r a Zen(-2) » a, is the Bohr radius. The most probable value of for an electron is: e {GATE 2008] (a) ay (b) 2a, (c) 4a, (d) 8a, ‘The normalized ground state wavefunction ofa hydrogen atomis given by y(r)=—L_-—2 6%, whereas Tae the Bohr radius and tis the distance of the electron from the nucleus, located at the origin. The expectation. 1 value (3) is: [GATE 2011] 8: 4 2 og OF OF A particle of mass nis; confined ina wo diuesiona square well potenti ofimensiona. This poeta V(x,y) isgiven by " . % The energy of the fiz a 2 [GATE 2012] wee ant? © oa? ma ray, fan a} Where a, isthe e enatom in the groundstate is ATE 2012] P60) @ PO) © 7a, Wa, An clectron in the ground state of the hydrogen atom has the wave function, 15. 16. 7. 18, 19. where a, is constant. The expectation value of the operator @= z?—,?, where z=rcos@ is (Hint Three Dimensional Potentials foe nan aE let! lo qn gm ) [GATE 2014} (a) 2 ad -2 (b) -a3 © @ 203 2 2 A patticle of mass ‘m’ is subjected to a potential, [GATE 2014) P(ny)=kma2(stay2), osese,-as ys The state with energy 4ha is g-fold degenerate. The value of g is A hydrogen atom is in the state IGATE 2014] where n,é,m in Ypeaf & a E Air) numbers, respectively. If Zi the angular momentul ; v ‘The normalized way respectively, the prin jin the mixed state Fn(2), where n, ¢ and m are, yely. Now consider an ebcixon ¥Y@)= $¥100¢ [TIFR 2012] ‘The expectation val Cag (eV) will be approximately @ 15 =(0)=3: (cyt Bi6cnee2 (A) -80.1 © +136 newry canna MWCO STEN deavour.com Ly? r vantibodmgtel +) sindcoroo| - +i] [TIER 2013] where a, is the Bohr radius. The principal (n), azimuthal (¢) and magnetic (m) quantum numbers corresponding to this wave function are (@ n=3,6=2,m=1 (&) n=2, © 1=3,0=2m=-1 @) n=2,¢=1m= #1 A rigid rotator is in a quantum state described by the waveftmetion =lm=1 ITIFR 2014) 1a u slals Hint: (a) tLis are, 2tron oly bers ‘Three Dimensional Potentiale 20. ai, 22, 23, Where @ and @ are the usual polar angles. Iftwo successive measurements of L, are made on ths rotator, the probebilty that the second measurement will yield the value +h is (@) 0.25 (b) 0.33, (05 @ negligible A particle in the 2s state of hydrogen has the wave function 1 (1y"(,_r) (_ a (r)= —| [2-4 Jexp|-- va) ant) [ a) Pl 4) where ‘ris the radial coordinate win. the nucleus as origin and a, is the Bohr radius, The probability P of finding the electron somewhere inside a sphere of radius Aa, centred at the nucleus, is best described by the graph ITIFR 2014 ah ~ . This is = [TIFR 2015} ant “ws carosrorvleavour com : a ‘The ground state has no nodes in the interior of the box. (b) This is the most convenient choice of the zero level of potential energy. (©) Position and momentum cannot be exactly determined simultaneously. (@ The potential at the boundaries is not really infinite, but just very large, Consider a particle of mass ‘mn’ moving inside a two-dimensional square box whose sides are described by the equations x=0, x = L, y=0, y = L. Whatis the lowest eigenvalue of an eigenstate which changes sign under the exchange ofx and y? (TEST 2012] v 3At Sh OF © saF Ose © 5a ‘The binding energy of the hydrogen atom (electron bound to proton) is 13.6 eV, The binding energy ofpositro- nium (electron bound to positron) is [JEST 2012] (@13.6/2eV (b) 13.6/180eV (©) 13.6x1810eV._(@) 13.6x2 eV 24, 25, 26. a Three Dimensional Potentials Ifa proton were ten times lighter, then the ground state energy of the electron in a hydrogen atom would have been [JEST 2013, 2014] (a) Less (b) More (c) The same (d) Depends on the electron mass Consider a quantum particle of mass ‘m’ in a three-dimensional isotropic simple harmonic potential 1 V(x,y,2)= a. Find the ground state energy ofthe system. What would be the ground state energy ifthe particles are fermions. ‘The energy eigenvalue of a particle in the infinite square well is given by tee ma?” As the particles are bosons, all the N particles will be ia the = 1 state in the ground state configuration. Hence, the ground state energy of the configuration will be Nw © 2ma* Ifthe particles are fermions. a state can have only two of them. one snin nn and the ather enin dum n=1,2,3.. Ss icles nber und tion, Identical Particles = Therefore, the lowest N/2 states will be filled. The total ground state energy will be ant wee Ease. +(N/2)'] ay? \ eel aia a%ei) ma 6L2(2° er er Ama’ Example 2. Consider two noninteracting electrons described by the Hamiltonian = M(N+1)(N+2) Pye Ha Pes PHM) where V(x)=0 for 0a. Sfboth the electrons are in the same spin sat, what is the lowest energy and eigenfunction of the two-electron sytem? San, As both the electrons of spin part ofthe wavefunctions be [1 or | total wave functio /0 electrons (fermions), then the function must be antisymmettic. ate energy will corresponds to mbination for the space part is PCY SS por ee 0 and LS Oe PCH — 0, for O =1,2,3,.. 2ma ae ‘As two electrons can go into each of the states n = 1,2,3,..., the highest filled level will have n=8 and its energy will be nS? _ 32mm? Soln. Soin, Identical Particles (ii) The energy of the highest filled state is the Fermi energy E,. Hence, 2A ma* Example 4. What is the ground state energy and wave fiction for two identical particles in the potential defined in problem if the two particles are (i) bosons, and (ii) fermions? (i Bosons: Both the particles can be in the same state, Hence, ground state will correspond to 1, = 1,71, =1 and the corresponding space part will be symmetric in nature. The energy and wave function of the combined system are E ee aa (I) +£,(2)= ‘The spin part of the wave furiction of the two particle system must be symmetric. The wave function of the two particle system including spin is, 12 her spin down. Therefore, the nergy and wave functions are ees #119] rma Example 5. Consider fy Of three energy levels A, B and CC having energies E, te (A, and A,) and the system is in thermal equilibria} nding energy in the following cases: i ( the particles obey Fett terme om BARE EQDEAY iil) the particles are distinguis ‘ltzmann statistics... Denote the two states yeneraresreheeaverneunr: 3E by B and C, respectively. Ifparticle 1 is in A, and particle 24m X,the-conhgiratiortis marked as (A,, A). Thus, the symbol (B, C) indicates that one particle is in B and the other is in C. (i If the particles obey Fermi statistics, the system has the following configuration and energy. Configuration: (A.A), (A,B) (Ay B) (Ay ©, (Ay ©, BO) Energy: 28 3B 3B 4 4E SE (ii) 1f\8e particles obey Bose statistics, the additional configuration: (A,, A,), (Ay A,), (B, B) and (C, C) are also possible. Hence the configuration and energy are (Ay, Ag) (App B), (Ay, BY, (Ay, ©) (Ay, ©), (B,C), (Aj, Ay), (Ags AQ), (B, BY, (C, ©), 2E, 3B, 35, 4E, 4E, SE, 2E, 2E, 4B, 6E (iii) Since the particles are distinguishable, the following configurations are also possible: Configuration: (A,,A,), (B.A), (BA), (GA). (CA CB) Energy: 2E, 3E, 3E, 4E, 4E, SE Ia. Sol les Identical Particles a intial 3 the sare band stem wing ively. 1a, 30 Soln. Sola. Example 6, A one-dimensional potential well has the single-particle energy eigenfunctions y,(x) and y,(x) corresponding to energies E, and E, for the two lowest states. Two noninteracting particles are placed in the well. Obtain the two lowest total energies of the two particle system with the wavefunction and degeneracy if the particles are (i) distinguishable spin-half particle, (i) identical spin half particles, and (iii) identical spin zero particles. (i) Distinguishable spin-half particles. The particles have spin = half. Hence the total spin S=0, 1 when S=0,M,=Oand when S= 1, M,= 1, 0, ~1. Let us denote the spin wave functions by the corresponding |SM,) . As the particles are distinguishable, the two particles can be in y, even when $= 1. The different ‘wave functions and energies are wiG4)¥,04)100), B, +E, = 2B, vie )viCe)|IM,), M,=1,0,-1, £, +B, = 26, 4 articles. Again the. ‘otalspin Sse or Speman site directions. The degeneracy is 1 + (i) Two identical spin, When $= 0, the two particles are ips be antisymmetric. The space isymmetric. The wave function of with energy E, + When S= 1, one g el 2. Hence, the symmetric and ina 1-D box of length ‘L’, Both EEE AD Tee finction? Saeaserendeavolion 242 sath E= 2 (uP +n})-= andl AWhat is the space part of the wave 2=>m=2 m= Sm=lorm 2mi? Since, V1 =V space *V spin where, Vepace = symmetric and V spin = symmetric as two bosons are of same spin, Therefore, the symmetric part of the wave function willbe 1 Vspace =yglv(2)+v 2.0] = Jp (Zon tain 2 882 in | Soln. Soln, Gq = Identical Particles ‘Example 8. Consider a wave function y =y (71,7) 7, fora fermionic system consisting of two spin 4 patticles. The spatial pert of the wave function is given by v(isA)= lt @re)+4a)ali)] ‘The spin part of the wave function will be 1 (@) yy (8 + Ba) ©) le ~ a) (©) (aa) @ (6B) ‘Given space part of the wave function is symmetric nature and Viotat = V space XV spin = antisymmetric , 1 therefore spin part of the wave function willanti-symmetric ie. Zspin = ye ~ Ba) CSIR PREVIOUS YEAR QUESTIONS Three identical spin-1 distinot energy levels The num, ber of ways this can bed : F ICSIR Dec 2013] ‘ eee [d)2 gnerate energy level = 2s + 1 a] indistinguishable. Ifwe include ihians) can be distributed in two Correct answeris (b) Fermians are quantum: the concept of indist non-degenerate disti The number of ways On changing the dire the concept of di with spin-}é can be distri eahee will be identical. Ifwe include wie i vospithe ‘ciltiod prs race geayour-ai Level 2level Level 2elevel level 2elevel sande Hitt EEE — Ilevel So icles spin enum 1013) a nelude in two clude articles Identical Particles Soin. The number of ways = 8 But quantum identical particles are in general indistinguishable and there is no hint to consider particles distinguishable Correct answer is (a) Consider a system of two non-interacting identical fermions, each of mass inan infinite square well potential of width a, (Take the potential inside the wellto be zero and ignore spin). ‘The composite wavefunction for: ‘the system withtotal energy ui {CSIR June 2014} sce Correct answer: iste www. careerendeavour, com Identical Particles PRACTICE SET Consider the wavefiinction y = y(Z,%) 7, fora fermionic system consisting of two spin-half particles. The spatial part of the wavefunction is given by [GATE 2012] w(isB) = pla) %)4,(6)+4.(5)4(8)) Where 6, and@, are single particle states, The spin party, of the wavefunction with spin states (3 (1 o(+4) snap (-4] should be 1 1 @ Fy lob+Be) o globe) (© ea @ BB The ground state and the first excited state wave functions of a one dimensional infinite potential well are y; and yo, respectively, When two spin-up electrons are placed in this potential, which one of the following, with x, and xy-denoting:the:p roreleotronssearrgctly represents the space part of the ground state waverfifg [GATE 2014] ' i @ sluice 1 (%2)¥2(%)] 1 ie @ Flav l e ; (=2)v2(a1)] Which one of the follg g IGATE 2014] (a) a—particle {d) Deuteron ‘The spatial part ofa tw, ang |¥) represent the spin-up and spin-dawm states respee gel He state is - (JEST 2012] lilt) coy) Snack ea ‘Two electrons are confined in & one dimensional box of length L., The one-electron states are given by vy, (x) = W271 sin nex /L). What would be the ground state wave function y (x,,X,) ifboth electrons are arranged to have the same spin state? (JEST 2013] mk) (2k 28 (2) (2 sio{ 2) «2 sin( 28 si 4) 2. (mx,) . (2mx,)_ 2. ( 2nx, mx; ) ae ino 2. 1 \sin{ 22 2 sl ts L ) mal L : ( Li) ey 2... (2mm) t-— Si Perturbation Theory Soln. 7.1. Time Independent perturbation theory : ‘The potential energy of most of the real systems are different from the standard potentials that have been considered till now, and an exact solution isnot possible Different approximate methods have been devoloped to obtain the approximate solutions of those systems. One such method is time-independent perturbation, can be written as where 1, is the u ‘eigenvalues E? (n= 0,1,2. and correspondin oe to be known, The functions y! (n= 0,12...) the time independent perturbing Hamiltonian, “ee First order correcti First order comes f “ARES f, FE NDEAYV OME), www.careerendeavour.com _. ° Second order correction to energy of the n® state= £(°) = > Kvn || |ve) Eo ~ Ee Example 1: A particle of mass m, and charge ‘e" oscillates along the the x-axis in a one-dimensional harmonic potential with an angular frequency @. Ifan electric feld « is applied along the x-axis, evaluate the first and second order corrections to the energy of the ri state. ‘The potential energy due to the field 6 = ~B.E = ~eex ‘The perturbation H'= esx = ee, ay (a+at) Fist oder correctionto energy El!) = ce, 1 (nl(a a (a+at|n )=0 = mae

You might also like